MBE Torts

Ace your homework & exams now with Quizwiz!

A private nuisance is a thing or activity that substantially and unreasonably interferes with plaintiff's use and enjoyment of his land. The nuisance must be offensive, inconvenient, or annoying to an average person in the community. A plaintiff cannot, by devoting his land to an unusually sensitive use, complain of a nuisance based on conduct that would otherwise be relatively harmless.

A businessman operates a small child-care facility on his land. He often lets the children run around in the front yard. His neighbor is an owner of a dog, and the neighbor often lets his dog play around in his front yard. Some of the children in the child-care facility are afraid of the dog and they ask their parents to move them to another facility. The businessman has also noticed that many neighbors veer to the other side of the street rather than walk past the dog in the yard. After losing several customers, the businessman decides to sue the neighbor for nuisance.Which of the following is correct? The correct answer is:The neighbor is not liable for damages for private nuisance, because the neighbor's activity is not offensive or inconvenient to the average member of the community. Discussion of correct answer:A private nuisance is a thing or activity that substantially and unreasonably interferes with plaintiff's use and enjoyment of his land. The nuisance must be offensive, inconvenient, or annoying to an average person in the community. A plaintiff cannot, by devoting his land to an unusually sensitive use, complain of a nuisance based on conduct that would otherwise be relatively harmless. Here, the neighbor lets his dog play around in his front yard. If it was not for the businessman's child-care center and his policy of letting children play in his front yard, the neighbor's activity would be harmless. Therefore, the neighbor will not be liable for private nuisance. Incorrect. The neighbor is liable for damages for private nuisance, because the neighbor's activity substantially and unreasonably interferes with the businessman's use of his land. The neighbor is not liable for private nuisance because the neighbor's activity is not offensive to the average member of the community. Most people would not be bothered by this activity. It is only the particular use of the businessman's property that makes the dog in the neighbor's front yard a problem. Therefore, he will not recover for private nuisance.

Unlike other intentional torts, trespass to chattel requires actual damages. Taking a bike out for a spin and returning it before it is missed and still in the same shape is not damage grounds.

A college dormitory has a bike rack outside the front door. A college freshman woke up late on the day of his chemistry final examination. As he exited the dormitory, the freshman decided to take a bike owned by another student to get to class because if he walked to class it would take another 30 minutes. The bike did not have a lock. After he finished his chemistry final, the college freshman returned the bike exactly where he found it in the bike rack. The owner of the bike slept through the entire incident and never knew his bike had been taken.Does the owner of the bike have a cause of action for trespass to chattels? The correct answer is:No, because the owner of the bike has not sustained any damages. Discussion of correct answer:Trespass to chattel requires an intentional act that interferes with the plaintiff's possession of the property. The interference can amount to a temporary dispossession or something short of dispossession such as intermeddling, which does not directly affect the plaintiff's possession. Letting the air out of a car's vehicle would amount to intermeddling. Unlike other intentional torts, trespass to chattel requires actual damages. The facts indicate the college friend did temporarily dispossess the bike owner of possession, but the facts also indicate that there was no actual harm that resulted. It does not appear any harm was done to the bike nor does it appear that the bike owner was otherwise damaged. The bike owner was asleep during the dispossession and unaware the bike had been taken. While the bike owner will be able to demonstrate there was interference with his possession, he will be unable to prove any actual damages were sustained. Incorrect. Yes, because the college freshman intermeddled with the owner of the bike's possession of the bike. This answer is incorrect because while the college freshman will need to prove either interference or intermeddling, he will not be able to prove actual damages. The facts support a conclusion that there has been a temporary dispossession, but the bike owner will need to prove all of the elements, which includes proof of actual damages. The bike was not damaged and there are no facts to show he sustained any other type of loss.

If it's stupid and dangerous, you won't be excused just because it's tradition and you volunteered to help do it for someone else. If it ends up hurting someone, liable.

A fraternity president lived in a fraternity house with 25 other fraternity brothers. The fraternity had been known for years for its weekly wild keg parties. One of the better known features of the party was that at the end of the party all of the empty beer kegs were stacked up one on top of the other as evidence of the fraternity brothers' drinking prowess. This was a fraternal tradition. Every new fraternity brother was required to help the president stack the empty kegs. This week a new brother accepted the honor of stacking the kegs. The next morning the kegs came crashing down on a mailman who delivered mail to the fraternity house, breaking several bones in his foot. The mailman sued the new brother.Which of the following is correct? The correct answer is:The brother should be held liable, even though he was acting in a manner consistent with a fraternity custom that had gone on every week for years. Discussion of correct answer:Although acting according to a custom can be a defense to a charge of negligence, it is not conclusive. The trier of fact may find either that the customary manner of behavior was not reasonable under the circumstances or that a reasonable person would not have engaged in customary behavior. A trier of fact will probably find both that the custom of stacking kegs on top of one another was not reasonable and that a reasonable person would not engage in such dangerous behavior. Discussion of incorrect answers: Incorrect. The brother should not be held liable, because he was acting consistently with a fraternity custom that had gone on every week for years. The brother will still be held liable, even though he was acting according to the fraternity custom, because acting according to tradition is not always a defense to a claim of negligence. In this case, the custom was itself dangerous, and a reasonable person would not engage in such a potentially dangerous act. Thus, this answer is incorrect.

In order to be liable for battery, the defendant must act intending to cause a harmful or offensive contact. Hitting as part of an accident has no such intent where tortious battery is concerned.

A girlfriend and her boyfriend were third-year law students. They lived together in an apartment building. One evening, they got into a heated argument and after the boyfriend slapped the girlfriend, she left the apartment vowing never to return. As she was hastily walking down the hallway toward the front door, he chased her and begged her to stay. When she refused, he grabbed her arm and tried to apologize.The girlfriend then told the boyfriend the relationship was over and to please let go of her arm. The noise from this struggle attracted the attention of a neighbor. The neighbor opened her apartment door and stood in the hallway watching the girlfriend and the boyfriend grappling with one another. The neighbor then started shouting encouragement to the girlfriend. As the struggle ensued, the girlfriend swung her pocketbook at the boyfriend. When he ducked to avoid being hit, he fell backward into the neighbor and accidentally struck her chin with his elbow. The force of the blow fractured the neighbor's jaw.If the neighbor asserts a battery action against the boyfriend, will she prevail? The correct answer is:No, because the boyfriend struck her accidentally. Discussion of correct answer:Note that choices (A) and (B) are incorrect because in order to be liable for battery, the defendant must act intending to cause a harmful or offensive contact. Here, the boyfriend did not act intending to strike the neighbor. Therefore, the boyfriend is not liable for tortious battery. Discussion of incorrect answers: A) Incorrect. Yes, because an offensive touching occurred.

A private nuisance is an activity or a thing that substantially and unreasonably interferes with a plaintiff's use and enjoyment of his property. Unreasonableness is assessed by weighing the gravity of the harm against the social utility of the activity. So mentally unstable neighbors occasionally going out to sing or make noise for several minutes isn't enough.

A group home for persons with disabilities began operating in a residential neighborhood. The group home was in full compliance with all zoning requirements. Residents of the home were young adults who were being transitioned to independent living. On occasion, one of the residents in the home would have a panic attack and when he did, the group home coordinator calmed him by having him sing very loudly on the front lawn for several minutes. A neighbor of the home has filed an action for damages, alleging that the group home has caused the value of his home to decrease.Will the neighbor succeed in his action for damages? The correct answer is:No, because he does not have a claim for private nuisance. Discussion of correct answer:A private nuisance is an activity or a thing that substantially and unreasonably interferes with a plaintiff's use and enjoyment of his property. Unreasonableness is assessed by weighing the gravity of the harm against the social utility of the activity. There are no facts to conclude that there has been either a substantial interference or an unreasonable interference. The group home is in full compliance with zoning requirements and the resident's singing does not substantially interfere with the neighbor's use and enjoyment of his property because the facts indicate his singing occurs only occasionally and only for several minutes. The facts do not support a conclusion that the neighbor has been harmed in any way. Discussion of incorrect answers: Incorrect. Yes, because he has a claim for private nuisance. This answer is incorrect because the facts do not support this conclusion. Private nuisance requires a substantial and unreasonable interference with a plaintiff's use and enjoyment of his property. Substantiality is satisfied if the activity or thing is offensive or annoying to an average person in the community. Unreasonableness is assessed by weighing the gravity of the harm against the activity's social utility. There are no facts to indicate that the group home was offensive or annoying. It was in full compliance with the zoning laws. The resident's occasional singing would not meet the standard of substantiality. There are no facts showing any harm to the neighbor and thus the operation of the group home is not unreasonable.

First, misrepresentations per se does not exist. Second, to establish a prima facie case of intentional misrepresentation, a plaintiff must prove that the defendant's representation was false and made with scienter.

A hardware store owner sold power tools. A salesman for a certain brand of power tools came to the store and convinced the owner to discontinue his relationship with his current tool company and to exclusively carry his company's tools. The salesman told the owner that his tools were more powerful and therefore better than the other brand. He told the owner that a television handyman had recently signed a contract to serve as their celebrity spokesman and gave the owner a large sign to hang in his store advertising his power tools as the only tools good enough for the celebrity handyman. Relying upon the salesman's representations about his tools and the handyman, the owner bought the tools, hung the sign in the store, and removed the other brand of tools from his sales floor. Two weeks later, the owner discovered that the entire line of his new power tools had been recalled, because their motors were not powerful enough to qualify as power tools within the government's definition of the term. In fact, the motors were less than 25% as powerful as the other brand of tools the owner had removed from his shelves, a fact of which the salesman had been aware long before he met the owner. The owner also received a summons and complaint from the celebrity handyman's attorney who had filed suit for invasion of privacy, because the handyman actually was a spokesman for another brand of power tools. The owner filed suit against the salesman for misrepresentation.Who will prevail? The correct answer is:The owner, because he was justified in his reliance on the salesman's representations. Discussion of correct answer:To establish a prima facie case of intentional misrepresentation, a plaintiff must prove that the defendant's representation was false and made with scienter (knowledge of its falsity or reckless disregard of the truth), and that the plaintiff materially and justifiably relied on it and suffered damages. Reliance on a misrepresentation of fact is generally regarded as justified, except where the representation is patently false. The plaintiff has no duty to investigate, even where it would not be burdensome to do so. If the plaintiff actually investigates, the plaintiff may not rely on representations inconsistent with what he actually or might reasonably have discovered. Here, because all of the elements of misrepresentation are present, the owner will succeed because his reliance on the salesman's statements was reasonable. Discussion of incorrect answers: Incorrect. The owner, because the failure to meet government specifications makes the salesman's statements misrepresentations per se. To establish a prima facie case of intentional misrepresentation, a plaintiff must prove that the defendant's representation was false and made with scienter (knowledge of its falsity or reckless disregard of the truth), and that the plaintiff materially and justifiably relied on it and suffered damages. There is no such legal principle as per se misrepresentation. Because this answer relies on a principle of law that does not exist, it is incorrect.

Getting cut by your illegal chainsaw in a rescue attempt, however contrived, is of an injury type the statute was intended to prevent and you're among the class of persons intended to be protected by the statute, and thus your violation is not excused. The rescuer will win.

A hiker was hiking through the national forest early one morning when he came to an area where private citizens were permitted to cut their own firewood from dead trees which the forest service had designated for removal. A vacationer was the only person in the area, and was gathering up some logs he had cut with his chain saw when a large, dead oak tree was suddenly pushed over by a strong wind. The tree toppled onto the vacationer, pinning him underneath. The hiker ran to where the vacationer lay under the tree and saw that he had no chance to move the heavy oak. The vacationer, who had not been immediately killed because a fork of two huge branches had kept the full weight of the tree off of him, was nevertheless going to be slowly crushed or asphyxiated as the heavy tree slowly pressed him into the soft earth. The hiker grabbed the vacationer's chain saw, started it, and began cutting at the two large branches of the oak. He had almost cut the vacationer free when, because the chain saw lacked a chain brake which would instantly stop the chain's movement, he cut deeply into the vacationer's arm, nearly severing the arm. The hiker pulled the vacationer from underneath the tree, applied a tourniquet to the vacationer's arm, and summoned medical help. The vacationer's arm was saved after extensive surgery but he suffered a permanent loss of mobility.A state statute makes it illegal to own, possess or operate a chain saw without a chain brake that will instantly stop the motion of the chain when activated. The vacationer brought an action against the hiker to recover for the injury to his arm and the permanent disability which resulted when the hiker cut it with the chain saw. Assume the jurisdiction in question has not adopted any form of comparative negligence.What should be the outcome of this litigation? The correct answer is:The hiker should win, because the chain saw had no chain brake. Discussion of correct answer:The hiker should not be liable to the vacationer for injuring his arm with the chain saw. As to the effect of the hiker's violation of the safety statute, that violation is clearly excused, since the hiker had no way of freeing the vacationer from being crushed by the oak tree without using the chain saw. If violation of a statute is excused, that violation will have no mandatory effect on the determination of whether or not the violator was negligent. In addition, the vacationer is himself liable of contributory negligence per se, since he also violated the statute by owning and possessing a chain saw without a chain brake. Since his injury is the type the statute was intended to prevent, he is among the class of persons intended to be protected by the statute, and his (the vacationer's) violation is not excused. The vacationer's violation has the mandatory effect of conclusively establishing that he was contributorily negligent. The hiker was acting under emergency circumstances when he used the chain saw; no such emergency justifies the vacationer's ownership of the same saw. Incorrect. The vacationer should win, if a reasonable person in the hiker's position would have realized that the chain saw had no chain brake. It is immaterial whether the hiker realized that the chain saw had no chain brake. Ignorance would not alter the fact that the statute was violated. Under these circumstances, violation of the statute was excused, and the hiker was not negligent.

When an injured plaintiff sues a product manufacturer in strict liability, all she needs to show in order to prevail is that the product was defective and that the product was not substantially changed in the course of its distribution from manufacturer to consumer. Misuse of a product can be a defense to strict liability in tort, but slight deviations won't do.

A homeowner owned a backyard above-ground swimming pool. One day, she noticed that the pool filtration unit did not seem to be functioning properly. She determined that she needed a new filtration unit, so she went to the local pool supply store to purchase one. The homeowner looked for the model number for the filtration unit designed for her particular brand of pool, but noticed the store was out of that model. The homeowner saw another filtration unit that looked similar to the one she currently had. Although it was not manufactured specifically for her brand of pool, the homeowner decided it was close enough, so she purchased the filtration unit and took it home.The homeowner installed the filtration unit, carefully following the manufacturer's directions. When the installation was complete she activated the filtration unit. Because of a defect in the electrical system, the filtration unit shorted out, causing a spark that burned the homeowner's hands.In an action by the homeowner against the manufacturer, who will prevail? The correct answer is:The homeowner, because she was injured by the defective electrical system. Discussion of correct answer:When an injured plaintiff sues a product manufacturer in strict liability, all she needs to show in order to prevail is that the product was defective and that the product was not substantially changed in the course of its distribution from manufacturer to consumer. Because the electrical system was defective, and there is nothing in the facts to suggest that the pool filtration unit was substantially changed before the homeowner purchased it, the homeowner can sue the manufacturer under a strict liability theory. The fact that it may not have been the best unit for her pool is not relevant. Incorrect. The homeowner, because misuse is never a defense to a product liability action sounding in strict liability. This statement is too broad. In fact, misuse of a product can be a defense to strict liability in tort. However, foreseeable misuse is within the risks the manufacturer must plan for and is not a defense. Because misuse may be a defense to a strict product liability claim, this answer choice is incorrect.

Using a product you know is defective only to voluntarily and unreasonably continue use of the product is usually regarded as assumption of the risk and bars the plaintiff's recovery under Strict Liability.

A man bought a barbecue grill from a local store. The store only sold barbecue grills and did not manufacture them. One week later, the man noticed that the hose that ran from the propane tank had worn thin and the grill did not heat up as quickly as it should. The man was annoyed at this development, but he had a backyard full of hungry guests and continued to cook. One hour after he first noticed the worn hose, the man began to smell propane. He continued to cook, because he had only one hamburger left on the grill. Just as the man was finishing, the hose burst, and propane began rushing out of the tank. The propane ignited, seriously burning him.Will the man prevail in a strict liability action against the store? The correct answer is:No, because the man knew that the hose was worn. Discussion of correct answer:Strict products liability is invoked against manufacturers, suppliers, and distributors when a defective product for which defendant is responsible injures an appropriate plaintiff. Factors that govern whether strict liability applies include the defective nature of the product, the nature of injuries suffered, whether the parties to the suit are proper, and whether any defenses are available. However, if the plaintiff discovers or is warned of the dangerous defect and voluntarily and unreasonably continues to use the product, such continued use is usually regarded as assumption of the risk and bars the plaintiff's recovery in strict liability. Thus, if the man voluntarily continued to use the grill with knowledge of the danger, he will be barred from recovery in strict liability. Incorrect. Yes, if the defect in the hose could have been discovered by a reasonable inspection by the store. Strict products liability is generally imposed upon defendants who are in the business of manufacturing, distributing, or supplying the defective product. Negligence issues do not apply in strict liability cases. Thus, if strict liability applies, the store would be liable even if it had not been negligent in its inspection of the grill. As such, this answer is incorrect.

The minority rule is that a land occupier owes a duty of reasonable care to entrants on her land regardless of their status be they invitee, licensee, or trespasser. If none of the answers state the majority rule then this is likely the answer.

A man bought an elaborate remote control model airplane for his son's ninth birthday. The plane had a three-foot wingspan and was capable of every maneuver a real stunt plane could negotiate. The man instructed the son at length about use of the plane and only permitted him to use it under the man's direct supervision at a nearby field set aside for the use of model plane enthusiasts.One afternoon school was let out unusually early. The son headed home but was terribly bored, and disregarding his father's instructions, removed the plane from its cabinet in the garage and began playing with it in his front yard. With the plane soaring in circles above his house, the son became confused by the controls. He sent the plane peeling down the street at about a five foot height. A pedestrian on the adjoining sidewalk panicked when she saw the plane headed directly toward her. Scrambling into a homeowner's yard in an attempt to escape the plane, she fell over a large ceramic sleeping deer, breaking her hip.If the pedestrian sues the homeowner for the broken hip, who will prevail? The correct answer is:The pedestrian will prevail if the sleeping ceramic deer is an unreasonably dangerous condition which poses unreasonable risks to trespassers. Discussion of correct answer:A land occupier generally owes no duty to an unknown trespasser for risks posed by artificial conditions on his/her property. If one of the choices for this question stated this rule, that would be the best answer. However, there is no such choice here. The next best answer, then, is likely to be the minority rule. The minority rule is that a land occupier owes a duty of reasonable care to entrants on her land regardless of their status (invitee, licensee, or trespasser). Therefore, this answer is correct. Incorrect. The homeowner will prevail, because she owes no duty to persons on the public sidewalk. This answer tests your ability to read the facts carefully. The pedestrian was not on the public sidewalk at the time she was injured, but was injured on the homeowner's land when she fell over a ceramic deer in the homeowner's yard. Therefore, this answer is incorrect.

A landowner is strictly liable for personal injuries--even to unknown trespassers--inflicted by vicious watchdogs, as distinct from other animals with known dangerous propensities. It's the same as saying you can't leave bear traps in your house to ward off robbers. Besides, the man here was a rescuer. It is not relevant whether the owner was negligent.

A man was walking along a city street when he heard cries for help coming from inside a fenced yard. The man noticed a sign on the outside of the fence that said: "Beware! Vicious Attack Dog." The man peered over the fence and saw a boy cowering in a corner of the yard as a large snarling dog approached him. The boy had climbed into the yard on a dare from one of his friends, even though he knew the dog in the yard had a reputation for attacking strangers who entered. However, as the boy climbed over the fence he slipped and severely sprained his ankle so that he could not climb back out of the yard. The man quickly realized that the boy needed help or he would be mauled. The man jumped into the yard and distracted the dog as the boy crawled to a gate. The dog quickly turned and ran at the man, as the boy opened the gate, crawled out, and closed it behind him. The dog jumped at the man just as he leaped out of the yard, severely scratching the man's left leg. The owner had placed signs all around her property warning of her dog because she knew the dog posed a danger to anyone who entered. The man sued the owner for his injuries.What will be the likely result? The correct answer is:Judgment for the man, because the owner is strictly liable for injuries caused by her animal. Discussion of correct answer:A landowner is strictly liable for personal injuries--even to unknown trespassers--inflicted by vicious watchdogs, as distinct from other animals with known dangerous propensities. This is because vicious watchdogs are considered to be equivalent to mechanical devices. Besides, the man cannot be considered a trespasser, because he was on the owner's land to rescue the boy and thus was privileged by necessity to be where he was. Discussion of incorrect answers: Incorrect. Judgment for the owner, because she was not negligent under the circumstances. Landowners are strictly liable for personal injuries inflicted by vicious watchdogs, even to unknown trespassers. This is because vicious watchdogs are considered to be equivalent to mechanical devices. Therefore, it is not relevant whether the owner was negligent. Thus, this answer is incorrect.

A parent may be liable for the intentional torts of a child. A parent has a duty to exercise reasonable care to control a child to prevent the child from intentionally harming others. However, it must be show that the parent had reason to know of the ability to control the child and the necessity to control the child.

A mother bought her 14-year-old son a new BB gun for his birthday. The next day the son decided to shoot at cars passing by the house. From his bedroom window, the son would aim at the hubcaps of the cars. The son saw a car slowly coming down the street that was driven by an elderly woman. The son aimed for the hubcaps but instead shot through the passenger side window. While the elderly woman was not hit, the noise of the glass breaking caused her to lose control of the car and crash into a telephone pole. The elderly woman brought a suit for damages against the mother.Will the elderly woman likely prevail in her suit for damages? The correct answer is:Yes, because a parent may be liable for the intentional torts of a child. Discussion of correct answer:A parent has a duty to exercise reasonable care to control a child to prevent the child from intentionally harming another person or from creating an unreasonable risk of harm to another person when the parent knows or has reason to know she has the ability to control the child and knows or should know of the necessity and opportunity to exercise control over the child. Under this standard, the mother may be liable for the son's intentional tort. Here, the facts indicate that the mother may face liability if the elderly woman can prove the mother knew or had reason to know of her ability to control the son and the necessity to control the son. C) Incorrect. No, because a parent is not vicariously liable for the negligence of a child. This answer is incorrect because it reaches the wrong conclusion under a rule that does not apply to the facts. While it is correct that a parent is not vicariously liable for the negligence of a child, a parent can be liable for her own negligence for failing to exercise reasonable care in controlling a child to prevent harm to others. Here, the facts may support a finding of negligence by the mother for failing to exercise reasonable care over the son in his use of the BB gun.

A defendant must show a privilege as a defense (such as right to self-defense), rather than the reason for why it happened that way (she never consented to tackle).

A neighbor, along with several others, is watching a neighborhood game of football. It is a neighborhood tradition that whenever a player scores a touchdown, the player will rush at and tackle one of the fans on the sidelines. Soon, a player scores a touchdown. The player immediately looks toward his brother, who is watching the game and standing directly behind the neighbor, and the player charges at his brother. The neighbor, believing that she is about to be bowled over, swings her purse at the player, breaking his nose. The player sues the neighbor, and the neighbor asserts the privilege of self-defense.Who will prevail? The correct answer is:The neighbor, because even when mistaken, the neighbor has a right to defend herself. Discussion of correct answer:Where a defendant is reasonably mistaken as to his or her need for self-defense, the privilege of self-defense still exists. Here, although the neighbor was not the actual target of the player's tackle, she reasonably believed that she was; as such, her defense will stand. Discussion of incorrect answers: Incorrect. The neighbor, because the neighbor did not consent to being tackled. While it is true that a plaintiff's consent to contact will negate his or her right to recover against a defendant in a battery claim, in this instance, the neighbor is the defendant in the player's battery claim, not the plaintiff. Thus, this answer confuses the parties and the issues at hand.

intrusion upon seclusion, an invasion of privacy tort requires the intentional intrusion upon the solitude of another and that the intrusion be highly offensive to a reasonable person. Unlike all other forms of invasion of privacy actions, a successful intrusion into seclusion action does not require publication.

A photographer learned that the mayor of his major city had made an appointment at the veterinary office where the photographer's niece worked. The photographer's niece told him that the mayor was planning to have his ailing 16-year-old dog euthanized. The photographer, an ardent supporter of the mayor, recognized this as a priceless opportunity to boost the mayor's re-election campaign. The mayor was known to be smart and capable, but had been often been criticized as cold and unfeeling. The photographer was convinced that a personal piece about the politician and his dying dog would show the public that this was a candidate with a heart.The next day, the photographer lingered in the veterinary waiting room until the veterinarian came to lead the politician and his dog into a back room. Several minutes later, the photographer saw the vet exit the room, shutting the door behind her. The photographer then opened the door and began snapping pictures of the tearstained mayor stroking the dead body of his beloved pet. At the sound of the camera, the mayor looked up and yelled, "What the hell are you doing? Get out of here!" The photographer tried to explain that he was trying to help the mayor's chances of winning re-election, but the mayor was furious. Shocked at the mayor's reaction, the photographer fled the room. The incident left him so rattled that he gave up on writing the article. The mayor filed suit against the photographer for intrusion upon seclusion, an invasion of privacy tort. Is the mayor likely to prevail? The correct answer is:Yes, because the photographer's act would be highly offensive to a reasonable person. Discussion of correct answer:The tort of intrusion into seclusion requires the intentional intrusion upon the solitude of another and that the intrusion be highly offensive to a reasonable person. In this case, both elements are present. The photographer clearly intruded upon the mayor's solitude, and his intrusion would be highly offensive to a reasonable person. The situation was also one in which the mayor had a reasonable expectation of privacy. As such, the mayor is likely to prevail. Incorrect. No, because the photographs were never published. The tort of intrusion into seclusion requires the intentional intrusion upon the solitude of another and that the intrusion be highly offensive to a reasonable person. Unlike all other forms of invasion of privacy actions, a successful intrusion into seclusion action does not require publication. Therefore, it is irrelevant that the photographer did not publish his photographs.

Self-Defense, you can throw an apparent intruder in your home out the door and onto their ass. Valid if reasonable.

A pre-med student and a biology student lived together in a rental house off campus. During the semester break, the campus dormitories were closed, and students were not permitted to stay on campus. An education student, who lived on campus, asked the pre-med student if he could spend the semester break at his house. The pre-med student, who was going to visit his family during the break, agreed. The pre-med student gave the education student a key to the house and told him, "My room is yours." Then the pre-med student left town, leaving a note on the table for the biology student, informing him that the education student would be staying for the break.That afternoon, the biology student came home after class but did not notice the pre-med student's note. Late that night, the biology student was awakened by the sound of someone downstairs. He came down to investigate and discovered the education student in the hallway outside of the pre-med student's room. Before the education student had a chance to explain, the biology student seized him, opened the front door, and pushed him out. The education student fell and fractured his leg.Will the biology student face liability for forcibly removing the education student from the house? The correct answer is:No, because the biology student can successfully assert self-defense. Discussion of correct answer:Even where there is actually no harm threatened against the defendant, he may successfully assert self-defense if a reasonable person in the same circumstance would have believed that he was under attack. Thus, so long as the defendant subjectively (i.e., honestly and in good faith) believes that a sufficient threat exists to justify defensive force, and there is an objective basis for that belief (i.e., a reasonable person would believe so under the circumstances), self-defense is available. Here, the biology student acted reasonably in response to a perceived harm (i.e., an unexplained intruder in his house in the middle of the night). The biology student subjectively believed that the threat was real (he had not read the note, nor did he actually think or believe anyone would be in the house besides his roommate); he also had an objective basis for the belief (a reasonable person in his situation would have also perceived the education student's presence as a threat). He met the stranger with non-deadly force necessary to protect himself and his home from intrusion. Therefore, he will be able to successfully assert self-defense, and will therefore not face liability for forcibly removing the education student (despite the education student's privilege). Incorrect. Yes, because the education student was privileged to be in the house. Where the plaintiff's conduct, which purportedly threatens an imminent attack, is privileged, a defendant may generally not invoke self-defense and will be liable for any tortious acts committed toward the plaintiff. However, this is superseded where a reasonable person in the same circumstance would have believed that he was under attack.

Intending to place reasonable apprehension of an imminent harmful contact upon someone, even by automatically triggered noises, is sufficient intent to support liability for assault.

A professor recently moved to a new home in a distant state. She decided to explore her new neighborhood on foot. Several blocks from her house, the professor saw what she believed was a footpath across public land. In actuality, it was a footpath that led onto private property. Over the years, the landowner had been annoyed by the number of people who used the footpath to enter onto his land thinking it was public property. For that reason he set up a device that contained a sensor that was triggered by movement on the footpath. When the sensor was activated, the device would emit a loud noise that sounded like a gunshot. The landowner's intent was to frighten anyone walking on the footpath into thinking they were being shot at so they would not return. When the professor walked along the footpath, she activated the sensor and the loud noise was activated. The professor was momentarily startled by the noise and stepped back, tripping over a tree root. The professor fell to the ground, suffering several bruises.Can the professor recover from the landowner for battery? The correct answer is:Yes, because the landowner intended to frighten anyone walking on the path. Discussion of correct answer:The landowner intended to place anyone walking along the footpath in reasonable apprehension of an imminent harmful contact. This is sufficient intent to support liability for assault. Generally, if a defendant intends to commit assault, battery, false imprisonment, trespass to land or to chattels, and inadvertently commits a different one of these other torts, his intent will "transfer" to support the tort appropriate for the result achieved. Therefore, because of transferred intent, the landowner is considered to have intentionally caused a harmful contact, which makes the landowner liable for battery. Incorrect. No, because the landowner did not intend to cause a touching. The landowner's intent was to frighten, which is sufficient intent for the tort of assault. Through the doctrine of transferred intent, that intent can be used to establish the tort of battery. The landowner is liable for battery, even though he did not intend to touch the professor. Thus, this answer is incorrect.

When the plaintiff has no practical alternative, due to the defendant's negligence, but to face a known risk, the plaintiff has not assumed that risk. For instance you're stuck in the mall and your child is with a babysitter, and you decide to climb out a second story window.

A salesperson and a customer were in a fitting room of an expensive clothing store for which the salesperson worked near closing time. To make sure they were not locked in, the salesperson told the head of security to check the fitting rooms before securing the store for the night. A few minutes later the head of security caught a shoplifter and was required to detain him until the police could arrive and take custody. The head of security then locked up the store and left. The salesperson and the customer were still in the fitting room. When the women finished a half hour later, they tried to leave by the front and then the back doors but both were locked. Upon attempting to telephone for assistance, the salesperson discovered that the phones were out of order. Neither woman had a cell phone. Because the store was the last to close in the mini-mall in which it was located, the women knew that no passers-by could be relied upon for help. When the customer insisted that she absolutely had to be somewhere in one hour, the salesperson led her to the second story office of the store manager. A window in that office opened onto a foot-wide ledge, and a few yards along the ledge a drainpipe provided a means of climbing to the ground onto the vacant field at the back of the mall. The salesperson warned that this route was dangerous, but the customer replied that she had to pick up her son from the babysitter. The customer then climbed out of the window and proceeded out onto and along the ledge. As she landed on the uneven ground after sliding down the drainpipe, the customer sprained her ankle. The customer brought an action against the store for her sprained ankle.Assuming that her theory of recovery is negligence, what should result if the defendant store pleads assumption of risk as its only defense? The correct answer is:Judgment for the customer, because she did not undertake the risk of falling voluntarily. Discussion of correct answer:Assumption of risk requires that a plaintiff be subjectively aware of the risk of harm and knowingly expose himself or herself to the danger. When the customer was trapped in the store through no fault of her own and had a pressing need to be elsewhere in one hour, her attempt to escape by climbing down from the window was not voluntary. When the plaintiff has no practical alternative, due to the defendant's negligence, but to face a known risk, the plaintiff has not assumed that risk. Incorrect. Judgment for the store, because the risk of injury from a fall was clearly foreseeable under the circumstances. The fact that a risk is foreseeable does not mean that a plaintiff has voluntarily consented to the risk, which is necessary for assumption of risk to apply. Where the plaintiff is compelled to undergo the known risk due to the defendant's negligence, there is no voluntary assumption.

Privilege of Competition: In the absence of prohibition by statute, illegitimate means, or other unlawful conduct, a defendant seeking to increase her own business may cut rates or prices, allow discounts, or enter into secret negotiations behind the plaintiff's back, refuse to deal with the plaintiff, or threaten to discharge employees who do.

A woman operates a large house cleaning service in a small city. The woman employs 25 people. She is very successful, but recently a national cleaning service chain opened a branch in the small city. The new branch offers the same service for much lower prices. The woman is not happy about this new branch, because she is afraid many of her customers will switch services. In order to decrease the potential pool of new customers for the new service, the woman announces a policy that she will discharge any of her employees if they become customers of the new branch. Later, one of her employees contracts with the new branch to have her house cleaned. The woman learns of her employee's actions and discharges the employee. The national cleaning service then sues the woman for interference with prospective advantage.Which of the following is correct? The correct answer is:The national cleaning service will not prevail, because the woman's actions were privileged. Discussion of correct answer:The tort of interference with prospective advantage protects the probable "expectancy" interests of future contractual relations of a party, such as the prospect of obtaining employment or the opportunity to obtain customers. The most common defense to the tort centers around the privilege of competition. In the absence of prohibition by statute, illegitimate means, or other unlawful conduct, a defendant seeking to increase her own business may cut rates or prices, allow discounts, or enter into secret negotiations behind the plaintiff's back, refuse to deal with the plaintiff, or threaten to discharge employees who do. Here, the woman threatened to discharge employees who deal with the new branch, so her actions will be privileged by competition. As such, the woman will not be held liable. Incorrect. The national cleaning service will prevail, because the woman interfered with the new branch's opportunity to obtain customers. Even though the woman interfered with the new branch's opportunity to obtain customers, the woman's defense will be that her action is privileged by competition. Thus, the woman will not be held liable. In the absence of prohibition by statute, illegitimate means, or other unlawful conduct, a defendant seeking to increase her own business may cut rates or prices, allow discounts, or enter into secret negotiations behind the plaintiff's back, refuse to deal with the plaintiff, or threaten to discharge employees who do.

Res Ipsa loquitur to apply, a plaintiff must establish three things. First, the event that caused the injury would not occur in the absence of negligence. Second, it is more likely than not that the defendant's negligence caused the event. Third, the plaintiff is not responsible for the event that caused the accident.

A woman was shopping at a department store. She got on the store's escalator on the third floor and was riding it to the fourth floor. The escalator came to a sudden stop, causing the woman to fall backwards. At the time the escalator suddenly stopped, the woman was not holding onto the handrail. She suffered several broken bones and injuries to her back. Apart from the sudden stop, nothing indicated that something was wrong with the escalator. A regular maintenance examination was performed on the escalator six days before the incident. After the incident, a maintenance specialist examined the escalator and found it to be in proper working order. The woman brought a negligence action for damages against the department store. At trial, the woman requested a jury instruction based on the doctrine of res ipsa loquitur. The trial judge gave the requested instruction, and the jury returned a verdict in favor of the woman. The department store has filed an appeal claiming the trial court erred by giving an instruction on res ipsa loquitur.Will the store likely prevail on appeal? The correct answer is:Yes, because the woman will not be able to demonstrate that the sudden stopping of an escalator is an event which would not ordinarily occur in the absence of negligence. Discussion of correct answer:The doctrine of res ipsa loquitur is available to help a plaintiff establish the defendant breached a duty of care when the circumstances make it impossible for a plaintiff to determine exactly what the defendant did or failed to do that gave rise to the injuries. When the doctrine applies, a jury is permitted to infer negligence. For the doctrine to apply, a plaintiff must establish three things. First, the event that caused the injury would not occur in the absence of negligence. Second, it is more likely than not that the defendant's negligence caused the event. Third, the plaintiff is not responsible for the event that caused the accident. While the woman will be able to show she was not responsible for the escalator suddenly stopping, she will not be able to prove either the first or second elements. The sudden stop of an escalator is unlikely to always be the result of negligence. Further, the facts demonstrate that the defendant was not negligent, having done an inspection six days before the accident and following the accident it was determined the escalator was in proper working order. D) Incorrect. No, because the woman will be able to demonstrate that it is more likely than not that the store's negligence caused the escalator to suddenly stop. This answer is incorrect because the facts do not support this conclusion. Here, the facts demonstrate that shortly before the accident the escalator had been inspected. And immediately after the accident, the escalator was examined and found to be in proper working order. These facts eliminate the possibility that the plaintiff will be able to prove that the event that caused the injury would not have occurred in the absence of negligence or that it is more likely than not that the defendant's negligence caused the event.

Without intention, where a person enters the land of another through negligence, recklessness, or as a result of an abnormally dangerous activity then in order to be liable for trespass he must cause damage to the land.

After winning a big antitrust case, an attorney and a few associates decided to celebrate. After having a few drinks, the attorney left his friends and drove home. The attorney, who was a bit tipsy, began driving in an erratic and reckless manner. He was traveling at an excessive speed along a residential section of town when he approached a sharp curve in the roadway. Trying to negotiate the turn, the attorney lost control of his vehicle, veering off the road and landing on the front lawn of a homeowner's property causing damage to the rose bushes.If the homeowner asserts a claim against the attorney for entering her land, will she recover? The correct answer is:Yes, because the attorney damaged her land. Discussion of correct answer:One is subject to liability for trespass irrespective of whether he causes harm to any legally protected interest of the other, if he intentionally enters the land in possession of the other. Based upon the given facts, the attorney did not intentionally drive his vehicle onto the homeowner's property. Rather he lost control of his vehicle while trying to negotiate a sharp curve in the highway. In this regard, the attorney may be liable for trespass because he was operating his car in a reckless manner. However, where a person enters the land of another through negligence, recklessness, or as a result of an abnormally dangerous activity, in order to be liable for trespass he must cause damage to the land. Therefore, choice (C) is a better answer than (A) because even though the attorney was reckless, he will not be liable unless he damaged the homeowner's property. Note that choice (B) is not correct because the attorney's entry was not intentional. B) Incorrect. Yes, because the attorney entered onto her property

First, read the hypo carefully. Second, if it was not an intended, malicious conduct and if not harm was suffered then neither punitive nor nominal damages may be acquired.

An avid collector's collection of Union and Confederate mortar shells was the world's largest collection of Civil War memorabilia. Feeling that he was obligated to share his piece of American history with the nation, the collector regularly loaned his collection to museums and historical societies for exhibits. The historical society of the collector's hometown borrowed the collection for an exhibit celebrating the contributions the city made to the country and the world. Volunteers from the community offered their assistance to the historical society, a not-for-profit organization, in setting up the exhibit and transporting the mortar shells from the collector's house to the exhibit hall. One of the volunteers used his pickup truck to move some of the shells. On the road from the collector's home to the hall, the tailgate of the volunteer's truck opened, spilling mortar shells onto the road. None of the shells were damaged, but one of them struck a parked car causing extensive damage to the vehicle. The transportation of the mortar shells was televised live on the local news, and the car's owner, who was watching the news, witnessed the near destruction of his car, a rare and antique automobile.If the collector brings a negligence action against the volunteer for allowing the mortar shells to spill onto the road, what will he be entitled to recover? The correct answer is:Neither nominal nor punitive damages. Discussion of correct answer:The elements of a negligence action are a duty of care, a breach of that duty, actual and proximate causation, and damages. In seeking to establish liability in negligence, the collector must prove that: the volunteer owed a legal duty to the collector; the volunteer's conduct breached that duty; the volunteer's breach of the duty was the actual and legal (proximate) cause of injury to the collector; and the collector suffered damages as a result of the volunteer's conduct. Here, given that the mortar shells were not harmed in the accident, the collector did not suffer any damages. As such, the collector is not entitled to recover. Incorrect. Nominal damages. Nominal damages are traditionally used to vindicate a right. In some intentional tort cases where harm is not an element of the cause of action, such as trespass to land, nominal damages can be recovered. Here, however, the collector has not filed an intentional tort action but a negligence action. Proof of damages is an element of the cause of action of negligence. Even nominal damages cannot be recovered unless there is some proof of actual harm. Given that the mortar shells were not harmed in the accident, the collector did not suffer any damages as a result of the volunteer's alleged negligence. As the collector did not suffer any damages, he cannot recover even nominal damages against the volunteer.

If the home seller did not construct the home and is merely an "occasional" seller, so she will not be held strictly liable for the defect that caused the plaintiff. Not to be conflated with a service provider or seller of goods.

An executive has been living with her family in a house for three months when her company transfers her to another city. She hires a broker, and the broker sells the house to a young businessman. Shortly after the young businessman moves in, he is hurt by a piece of the ceiling that falls on his head. The businessman sues the executive and the broker for his injury.Which of the following is correct? The correct answer is:The executive will not be strictly liable, because she did not construct the house. Discussion of correct answer:Most jurisdictions hold sellers of mass-marketed new residences strictly liable for defects in those homes. Commercial suppliers at all levels of the distribution chain (i.e., manufacturer, distributor, retailer) as well as commercial lessors, new home developers, and sellers of used goods are potential defendants. However, the executive did not construct the home and is merely an "occasional" seller, so she will not be held strictly liable for the defect that caused the businessman's injury. Incorrect. The broker will not be strictly liable, because she is a service provider. The broker will not be strictly liable because he was not involved in building the house. Also, it is arguable that the broker is not a service provider, but a seller of goods. As such, this is incorrect.

If the group defamed is sufficiently large, then there is no cause of action for slander in any single member of the group.

At a conference concerning the criminal justice system in a populous state, a prominent criminal defense lawyer, who has practiced in most of the district courts of the state, made a statement that the district judges of the state can all be bribed and that money can buy a defendant an acquittal. A district court judge who has never presided over a matter in which the lawyer participated has sued the lawyer for slander.Which of the following would be the lawyer's best defense? The correct answer is:There are 250 district court judges sitting in the courts of the state. Discussion of correct answer:A statement directed to a group or class of persons can be "of or concerning" a particular person only if the group or class is small enough for the statement to be reasonably understood as referring to the plaintiff, who is a member of the group, or the circumstances give rise to the conclusion that there is a reference to a particular member of the group. If the group defamed is sufficiently large, then there is no cause of action for slander in any single member of the group. Because 250 judges would constitute a very large group, this is the lawyer's best defense. Incorrect. The only persons at the conference were sitting district court judges of the state. This is incorrect; it suggests that the statement was not "published," because it was only communicated to the persons defamed and not to any third parties. Even though the judges as a whole were the persons defamed, if the remark is specifically applicable to one judge, making the statement to that judge's colleagues would constitute publication for the purposes of defamation. If the judges can sue at all, they must sue as individuals. Thus, in each judge's defamation claim against the lawyer, every other judge in the room would be a relevant third party.

the level of duty of care the defendant owed to the plaintiff is an important part of case results. As a paying customer of the Defendant's business, the plaintiff is an invitee who is owed a duty of reasonable care, including the duty to inspect the premises for potential dangers to invitees.

At a girl's request, her mother sent her to a theater arts camp for the summer. At the art camp, the girl and the other campers studied voice, dance and drama, and enjoyed the great outdoors. Another camp for budding photographers was only a half mile down the lakeshore from the art camp and within a week the girl had become close friends with a camper there. One night the camper sneaked into the girl's cabin to visit. She climbed into the bunk above the girl. They giggled for awhile, but quickly fell asleep. The bunks were hung from the wall by a chain at each end in two tiers around the walls of the cabin. The counselor noticed that an extra bunk was filled that night but never gave it another thought.At about 3:00 A.M., without warning, the upper bunk the camper was in collapsed on the girl, breaking the girl's arm. The corner of the bunk hit a nearby table, knocking over a large candle lantern which had been left burning as a night-light. The camper's pajamas burst into flames. The counselor dragged the girl and the camper from their bunks, but not before the camper suffered moderate burns. The cabin burned to the ground. The fire inspectors were never able to determine precisely why the bunk bed collapsed.If the girl sues the art camp, what will be the most likely result? The correct answer is:Judgment for the girl, because her parents paid for her to attend the camp. Discussion of correct answer:This question turns on the level of duty of care the camp owed to the girl. As a paying customer of the camp, the girl is an invitee who is owed a duty of reasonable care, including the duty to inspect the premises for potential dangers to invitees. Therefore, because her parents paid for her to be there, she is an invitee and should prevail in her suit. Incorrect. Judgment for the girl, because she was injured by an artificial condition on the camp premises. What is relevant here is the girl's status on the land, which determines the duty of care owed to her by the camp. Because her parents paid for her to attend, the girl is an invitee. A land occupier owes a duty of reasonable care to invitees, including the duty to inspect the premises for potential dangers. Here, judgment will be for the girl because she was an invitee.

Gardener and Hardware store sued in comparative negligence case for exploding product let in garden. What best describes defendants' liability?

Both the gardener and the hardware store are liable for the full amount of damages. Discussion of correct answer:Where two or more defendants acting in concert injure a plaintiff, or where two or more defendants, acting independently, injure a plaintiff and the resulting damages cannot be allocated to particular defendants, each defendant is liable for the entirety of plaintiff's injury. In such a case, the plaintiff can execute against each defendant for the total damages suffered (although the plaintiff may recover only the total amount of damages awarded). In jurisdictions that have adopted comparative negligence, joint and several liability is still available to plaintiffs who have been awarded damages. However, any defendant compelled to pay damages to the plaintiff greater than the percentage amount for which he was found responsible may usually obtain appropriate pro rata contribution from the other defendants. Incorrect. The gardener and the hardware store are each liable for their percentage of fault, as determined by the jury. This answer misstates the meaning of comparative negligence. In jurisdictions that use comparative negligence, joint and several liability is still available to plaintiffs who are awarded damages. Where two or more defendants acting in concert injure a plaintiff, or where two or more defendants, acting independently, injure a plaintiff and the resulting damages cannot be allocated to particular defendants, all of the defendants are liable for the entirety of the plaintiff's injury. The plaintiff can execute against each defendant for the total damages suffered, although she may only recover from any or all defendants an amount equal to the total damages awarded. Any defendant compelled to pay damages to the plaintiff greater than the percentage amount for which he was found responsible may usually obtain contribution for the pro rata shares of the other defendants. Thus, this answer is incorrect.

Unreasonable interference with the use and enjoyment of the plaintiff's land is a nuisance type claim. The activity that causes the unreasonable interference can be the result of intentional conduct, negligence, or abnormally dangerous activity. Recovery can be had in nuisance even if the defendant was not negligent, meaning you don't need res ipsa loquitur.

Electric utilities used to use a highly toxic chemical to insulate transformers. As an electric company phased out its old transformers, it drained the chemical into a large storage container in its plant. When the last transformer containing the chemical had been retired, all of the chemical was shipped to a large underground storage facility that the electric company had built in a rural area.The storage facility was built in accordance with specifications, was state of the art at the time that it was built, and was fully expected to contain the chemical indefinitely. When the chemical was stored and the facility sealed, it was not known that, over time, the chemical could corrode the metals of which the storage facility was constructed. The chemical eventually seeped out of the facility into the adjacent ground.Ten years later, a cattle rancher whose ranch was within five miles of the storage facility found that a number of his cattle were dying. He tested his well water and found that it contained a high level of the chemical, which no one besides the electric company had ever used in the area. Unable to find an economically reasonable substitute source of water, he shut down the ranch and sold the land for $100,000 less than its fair market value prior to his discovery of the contamination.If the rancher sues the electric company for the $100,000 of damage he suffered, what is the likely result? The correct answer is:The rancher will prevail, because the chemicals which contaminated his well and forced him to terminate ranching came from the storage facility. Discussion of correct answer:The basis of the rancher's suit is nuisance, the unreasonable interference with the use and enjoyment of the plaintiff's land. The activity that causes the unreasonable interference can be the result of intentional conduct, negligence, or abnormally dangerous activity for which a defendant is strictly liable. Therefore, this is the best answer. Incorrect. The rancher will prevail, because under the doctrine of res ipsa loquitur, the electric company must have been negligent in the siting, construction, or maintenance of the storage facility. This choice incorrectly implies that negligence is the only available theory for recovery. In fact, recovery can be had in nuisance even if the defendant was not negligent. For example, in this case, the nuisance involves the escape of a highly toxic chemical from the facility. Storing a highly toxic chemical would have to be considered an abnormally dangerous activity and also, by definition, unreasonable. Thus, where the interference with plaintiff's property results from the defendant's abnormally dangerous activity, the defendant is strictly liable. All that the plaintiff needs to prove is that it was the defendant's activity that caused the damage.

superseding intervening force vs reckless conduct. Also, read answer more carefully.

Even though he knew that the city had an ordinance forbidding the riding of skateboards on city sidewalks, a man was riding his skateboard on a sidewalk in the city when he collided with a woman as she was leaving a jewelry store. The woman fell to the ground in great pain. A passerby telephoned 911 for help and the paramedics quickly arrived. The paramedics determined that the woman had broken her leg. They put a splint on the leg and summoned an ambulance to transport the woman to the hospital. When the ambulance arrived, the woman was placed in the back. The paramedics told the ambulance driver that there was no need to rush to the hospital since she was resting comfortably. However, the ambulance driver had a bet with a fellow driver that he could handle more calls in one day. The ambulance driver knew he had to rush to the hospital to drop the woman off so he could pursue another call. He therefore drove very quickly to the hospital. The ambulance driver lost control at a sharp curve because he was traveling too fast. The ambulance overturned, injuring the woman's back.If the woman sues the man for her back injury, what will be the impact of the ambulance driver's actions? C) The correct answer is:The man will not be liable for the woman's back injury because the ambulance driver's actions were reckless. Discussion of correct answer:If a rescue effort is foolish or reckless, it is deemed to be the sole legal cause of any additional injury resulting if the injuries are divisible. The woman suffered a broken leg in the original collision. Her back was injured in the rescue. These are apportionable injuries. Due to the fact that the ambulance driver was reckless in driving the ambulance very quickly, he will be solely responsible for the woman's back injury D) Incorrect. The man will not be liable for any of the woman's injuries because the ambulance driver's reckless conduct was a superseding intervening force. The ambulance driver's recklessness would not supersede the man's liability for the woman's original injury. The ambulance driver would, however, be responsible for the back injury he caused. Therefore, this is not the best answer.

Derivative liability for third party negligence. In most jurisdictions, the owner of a commercial enterprise open to business, invitees cannot delegate responsibility for the maintenance of the public premises to an independent contractor. Also, note that the word "only" might make an answer toxic.

In an effort to boost declining customer traffic, a local mall installed a carousel. Because it was the only carousel for hundreds of miles, the mall owners hoped children would drag their parents to the mall. The mall hired a technician company to install, operate, and maintain the carousel. The plan worked beyond anyone's expectations as traffic in the mall increased 200%.One day, a five-year-old girl came to the mall with her mother. After her mother bought hiking boots, the girl got her promised ride on the carousel. As the girl was boarding the carousel, she tripped and fell against part of the mechanism that made the horses go up and down. Someone had neglected to replace the protective housing that normally covered the machinery.The girl's hand was caught in the moving gears. A mall employee was walking by the carousel when the girl fell and was the closest person to the scene. Had the employee grabbed the girl quickly, the girl would only have suffered superficial cuts and bruises. Instead, the employee stared motionlessly at the girl, mouth gaping in amazement. As a result of the delay, the girl's hand was pulled further into the gear mechanism and three fingers were broken.If the girl's mother files suit against the mall, who will prevail? The correct answer is:The girl, if an employee of the technician company was negligent in not replacing the engine housing. Discussion of correct answer:The mall hired the technician company, an independent contractor, to install, operate, and maintain their carousel. In most jurisdictions, the owner of a commercial enterprise open to business, invitees cannot delegate responsibility for the maintenance of the public premises to an independent contractor. If the contractor is negligent in the construction or maintenance of the carousel, the business will also be liable. Discussion of incorrect answers: Incorrect. The girl, but only if the mall employee failed to act as a reasonable person. The mall might be liable if their employee was negligent in failing to attempt to rescue the girl. However, this is not the only way the girl could prevail in a suit against the mall. She might also prevail against the mall based upon their derivative liability for the technician company's negligence. The problem with this answer choice is the inclusion of the words "only if." The mall employee's failure to act reasonably is not the only way to establish liability.

If the plaintiff is weakened physically or placed in a position of peril due to the consequences of the defendant's negligence and that peril arises, the defendant will be liable for the additional injury to plaintiff. Provided it is a reasonable outgrowth of the incident. So if you put someone in crutches and two weeks later they fall because of the crutches, you're still liable if it was foreseeable.

Late for an important meeting, a man was driving down the street at a negligently high rate of speed. As he approached an intersection, another car driven by a woman negligently entered the intersection. The woman had been talking on her cell phone at the time and failed to notice the light was red for cars traveling in her direction. Had the man not been traveling at a high speed he would have been able to stop in time to avoid hitting the woman's car. However, he collided with the woman's car in such a way that the two vehicles spun and hit a pedestrian, who was lawfully crossing the street at the time. The pedestrian suffered a broken leg. The pedestrian's leg was placed in a cast and he was forced to use crutches for the next several weeks. One evening, about two weeks after the accident, while trying to make his way down a long hallway at work on the crutches, the pedestrian fell and broke his wrist.If the pedestrian sues the man for the additional injuries he suffered in the second fall, will the pedestrian prevail? The correct answer is:Yes, because the fall of a person on crutches is foreseeable. Discussion of correct answer:If the plaintiff is weakened physically or placed in a position of peril due to the consequences of the defendant's negligence and that peril arises, the defendant will be liable for the additional injury to plaintiff. The pedestrian's attempt to walk down the hall is a natural consequence of the pedestrian being injured and forced to use crutches by the man's negligence. It is a foreseeable outgrowth of an accident. Incorrect. No, because the second fall was a superseding, intervening cause of the pedestrian's injuries. Viewed as an intervening event, the pedestrian's second fall is not superseding because it is not abnormal or unforeseeable.

If it is not slander per se then a plaintiff must prove special harm or special damage in order to recover. Here because Ron is not accusing Henry of committing a crime &instead is making the statement that Henry would have stabbed his mother if she had not been killed in an auto accident.

Ron made four statements to George about their friend, Henry, who is a physician. Henry found out about these statements and filed four separate slander actions against Ron.For which of the following statements must Henry prove special harm or special damage in order to recover against Ron? The correct answer is:"George, if I were you, I wouldn't let Henry visit your mother. Henry would have stabbed his own mother to death last week, if she hadn't been suddenly killed in an auto accident." Discussion of correct answer:Because choice (C) is not an example of slander per se, Henry would need to prove special damages in order to recover against Ron in defamation. Choice (C) is not slander per se, because Ron is not accusing Henry of committing a crime. Instead, Ron is making the statement that Henry would have stabbed his mother if she had not been killed in an auto accident. The other three answer choices provide students with examples of slander per se, wherein the plaintiff can recover damages without proving that he has suffered any special harm at all. The cause of action is complete when the plaintiff proves that such a type of slander has been published about him and that he has suffered "actual injury" as a result of the defamatory statement. It is important for students to note the four per se categories for either slander or libel. Sections 571-574 of the Restatement 2d, Torts provides that "an action for slander will lie without proof of special harm only where: (1) the words impute a criminal offense which, if committed in the place of publication: (a) would be punishable by death or imprisonment; or (b) regarded by public opinion as involving moral turpitude; (2) the words impute to the plaintiff a presently existing venereal or other loathsome and communicable disease; (3) the words impute to the plaintiff conduct, characteristics, or a condition incompatible with the proper conduct of his lawful business, trade, profession, or with his public or private office, whether honorary or for profit; and (4) the words impute to a woman unchastity. Incorrect. "George, if I were you, I wouldn't call Henry for treatment in an emergency. He's a drunkard, and once he wasn't able to treat Jimmy because he was too intoxicated."

A police patrolman is a public official to whom the Times v. Sullivan [376 U.S. 254 (1964)] standard applies. This is the best defense you have if you defamed one. However, there can still be damages to reputation, nominal damages, punitive damages, etc, if said officer was fired due to this.

Rusty Rutledge had been a member of the police force of the town of Calypso for eight years. One day, while he was sitting at his desk at police headquarters preparing an accident investigation report, Police Commissioner Jay Ellsworth approached Patrolman Rutledge and said, "Rusty, I'm very disappointed in you. Read this letter I just received." Rusty unfolded the paper he was handed and read:"Dear Commissioner Ellsworth:I am a concerned citizen of Calypso and request that Patrolman Rusty Rutledge be restricted from carrying a weapon to his residence at 1231 Marshall Lane. Mr. Rutledge has tormented neighbors with foul language, has displayed a violent temper, has threatened little children, and has kicked pets. His conduct is most disgraceful as a human being and also as a police officer.Sincerely,Roger Chase"Rusty immediately jumped up from his chair, banged on the desk and screamed, "I do NOT have a violent temper!" He left the room, slamming the door behind him.In a defamation suit by Rusty against Roger, what would be Roger's best defense? The correct answer is:Law enforcement is a uniquely governmental affair, and police officers have a substantial responsibility for the safety and welfare of the citizens. Discussion of correct answer:Roger will want to eliminate his liability for defamation or at least want to point to facts that will subject Rusty to proving a higher standard (i.e., the standard that applies to defamation of a public official). These are facts Roger would want to point out to support his claim that a police patrolman is a public official to whom the Times v. Sullivan [376 U.S. 254 (1964)] standard applies. Choice (D) is incorrect. There can still be damages to reputation, nominal damages, punitive damages, etc. Choice (B) is incorrect on the principle of law stated. The U.S. Supreme Court has stopped short of the conclusion that all public employees are public officials. Even if this were the case, that doesn't mean Rusty may not recover any damages. D) Incorrect. Since Rusty was not fired from the police force as result of the letter, there are no damages he may recover in a defamation action.

Telling someone you will burn their house if they do something is not assault because the threat is not immediate physical harm.

The correct answer is:Judgment for the gang member, because he did not threaten any immediate physical harm. Discussion of correct answer:An assault is a volitional act, done with the requisite intent, which causes the plaintiff to experience a reasonable apprehension of an immediate harmful or offensive contact. If the defendant threatens harm in the future or is discernibly unable to carry out the threat, there is no assault. Here, the gang member threatened that he would burn down the homeowner's home, with the homeowner inside--manifestly a harmful physical contact--but the threat was to do so at some unspecified time in the future. The gang member is, therefore, not liable for the intentional tort of assault. Incorrect. Judgment for the homeowner, if the gang member intended that the homeowner would experience apprehension of a harmful physical contact. Intent to cause apprehension of a harmful physical contact (or intent to cause such a contact) is an element of assault. In the present question, the element of immediate harm is missing, and thus no liability for assault can be found, even though other elements of the tort are present.

Tort of Appropriation doesn't apply to animals. Such action would only be possible under a clear copyright infringement.

The correct answer is:No, because Harvey was not a living human being. Discussion of correct answer:The tort of appropriation of likeness has not been extended to animals, corporations, or other non-human plaintiffs. Therefore, because Harvey was not a living human being, his owners will not be able to recover for appropriation of likeness. (Note that this would not bar recovery through other means, such as copyright or trademark infringement.) Incorrect. Yes, if the owners prove that the facility gained financially from the advertisements. Here, the boarding facility used Harvey's likeness in an ad to promote its new rabbit facility. Ordinarily, the plaintiffs would not need to show that the defendant profited from the appropriation. However, the tort of appropriation of likeness has not been extended to non-human beings, such as animals or corporations. As such, the owners of Harvey would not be able to recover for this tort.

Telling someone you will beat them up tomorrow if they do something is not assault because the threat is not immediate.

The correct answer is:No, because the man's words would not place a reasonable person in apprehension of an imminent harmful or offensive contact. Discussion of correct answer:An assault is an intentional act that causes the plaintiff to experience a reasonable apprehension of an immediate harmful or offensive contact. Assault requires an intent to cause a harmful or offensive contact. The man's words do not demonstrate an intent to carry out threatened harm or an offensive contact. To state a claim for assault, the neighbor must have been placed in apprehension of an imminent harmful or offensive contact. While the harmful contact need not be immediate in the sense of instantaneous, there cannot be any significant delay. This requirement is not met here because the man's words do not indicate he was presently or imminently going to cause any harmful or offensive contact with the neighbor. Incorrect. Yes, because the man's words would place a reasonable person in apprehension of an imminent harmful or offensive contact. This answer is incorrect because the facts do not support this conclusion. The man's words do not threaten imminent harmful or offensive contact. Absent a threat indicating he had an intent to cause a harmful or offensive touching, the neighbor cannot satisfy the requirement of being placed in apprehension of an imminent harmful or offensive contact.

Slander Per Se (oral statement published to third person) vs Defamatory Statement of Fact (opinion implying speaker knows certain facts exist or are true), if neither no case.

The correct answer is:No, because the statement was not a defamatory statement of fact. Discussion of correct answer:Slander requires a defamatory oral statement that is published to a third person. The statement must communicate a message or meaning that holds a person up to hatred, scorn, or ridicule, or causes customers to stop doing business with them. Generally, an action for defamation exists only for statements of fact, as opposed to an opinion. However, an opinion that implies the speaker knows certain facts to exist or to be true is actionable. Unless a statement is slanderous per se, damages must be proven. A statement is slanderous per se and thus damages are presumed when it imputes to the plaintiff the commission of a crime that involves moral turpitude or a type of infamous punishment, alleges the plaintiff has a loathsome disease, imputes dishonesty or incompetency or other improper conduct in the operation of a business or profession, or accuses a woman of being unchaste. The statement would not be actionable because it is not defamatory nor is it a statement of fact. The neighbor actually commends the quality of the auto mechanic's work but then opines he thinks he charges too much. This statement would not necessarily cause customers to avoid doing business with the auto mechanic nor would the statement otherwise hold the auto mechanic up to scorn, ridicule, or hatred. Besides not being defamatory, the neighbor is offering his opinion based on what he has heard rather than making a statement of fact based on his personal experience. Incorrect. No, because the statement was not slanderous per se. This answer is incorrect because while it is true that the statement was not slanderous per se, the auto mechanic's suit would fail because the neighbor's expression of opinion would not be actionable regardless of whether it carried a slanderous message. The neighbor's expression of opinion about the auto mechanic's high fees was not a statement of fact.

Violation of a statute only proves breach when it is an unexcused violation. A tourist that was not a local resident, & existence of school hidden by the hedge, and no sign existed informing of school district or 15mph rule. Only excused if driving as a reasonable person under the circumstances. Ignorance of law is excuse for civil, not crimes.

The correct answer is:No, unless the tourist should have been driving more slowly under the circumstances. Discussion of correct answer:The tourist violated a local ordinance requiring drivers to maintain a 15-mph speed limit on streets surrounding a school. The purpose of the statute is to protect school students, be they adult or otherwise, from serious injury from rapidly moving cars. However, violation of a statute only proves breach when it is an unexcused violation. Here, the tourist was not a local resident, the existence of the school was hidden by the hedge, and no sign existed informing the tourist that this was a school district. The tourist did not know that he faced a situation in which the statute would apply. A reasonable person in his situation would not have known to slow down to the 15-mph limit. Hence, the tourist's noncompliance will be excused if he was driving as a reasonable person under the circumstances. Keep in mind that ignorance of the law is no defense to a crime, but here we are not dealing with criminal liability, but rather civil tort liability. Discussion of incorrect answers: Incorrect. No, because the hedges prevented a driver on the street from seeing the school. Although the tourist might be excused for the violation of the statute because of the obscuring hedge, he could still be liable if he was driving unreasonably rapidly under the circumstances.

Borrowing a car precludes claim of wrongful interference unless incident occurred for unauthorized use. A normal lender would need negligence as a basis to recover anything at all.

The correct answer is:Nothing, because the damage to the cousin's car was not caused by negligence on the part of the man. Discussion of correct answer:At first glance, the facts of the question may appear to raise issues of trespass to chattels or conversion. The intentional tort of trespass to chattels involves an act by the defendant that interferes with the plaintiff's rightful possession of personal property. Conversion involves an act by the defendant that interferes with the plaintiff's rightful possession of personal property to such a degree that it is just to require that the defendant pay the plaintiff the full value of the property. Both torts require the defendant's wrongful interference with the plaintiff's possessory interests. However, under the facts of the question, the man borrowed the car with the owner's permission. The fact that the man used the cousin's car for an unauthorized purpose is irrelevant, because the accident did not occur while the man was engaged in this unauthorized use. Unless the man was negligent in some manner that helped cause the accident, such as in the way that he parked the car in front of his prospective employer's building, he is not liable for the damage to the car. Thus, this is the best answer. If the cousin sues the man for the damage to his car, he will most likely recover nothing, unless the damage to the cousin's car was caused by negligence on the part of the man. Discussion of incorrect answers: Incorrect. The cost of repairing the cousin's car, because the car was damaged while under the man's control. This answer choice describes the measure of damages that would apply if the man had been liable for trespass to chattels. In this case, the man is not liable unless the damage to the car occurred as a result of the man's negligence.

You owe a known trespasser a duty to warn of known conditions. Best answer.

The correct answer is:The boyfriend's presence had been discovered by the maintenance worker before he was injured by the goblin. Discussion of correct answer:Even though the maintenance worker may not have known that the boyfriend's presence was unauthorized, the boyfriend would at least have been either a discovered trespasser or licensee. In either status, the boyfriend would be owed a duty to be warned of known artificial conditions that are unreasonably dangerous and would be owed a duty of reasonable care to be protected from injuries arising from activities on the land. Activating the goblin could be considered an artificial condition or activity. The language in this choice describes the boyfriend as a discovered trespasser and yields the strongest set of facts for his recovery. Incorrect. An inspection by the maintenance worker would have revealed that the goblin could malfunction and injure passersby. The duty owed to known or anticipated trespassers extends to known dangerous conditions. There would be no duty to inspect. Therefore, this fact would not help the boyfriend.

legislator (federal or state or local) is not liable for a defamatory message uttered while on the floor of the legislature. Even if matter has nothing to do with legislature, immunity still.

The correct answer is:The friend is not likely to prevail, because the councilman's statement was privileged. Discussion of correct answer:A legislator (federal or state or local) is not liable for a defamatory message uttered while on the floor of the legislature or during hearings, committee proceedings, or the like. The nature or content of the defamatory message or its relationship to any matter before the legislature is immaterial to availability of this privilege. Even though the councilman's statement had nothing to do with city matters, the immunity still applies, and the friend will not prevail in his defamation action. Discussion of incorrect answers: Incorrect. The friend is likely to prevail, because the councilman's statement had nothing to do with the councilman's duties, nor any city matters. A legislator (federal or state or local) is not liable for a defamatory message uttered while on the floor of the legislature or during hearings, committee proceedings, and the like. The nature or content of the defamatory message or its relationship to any matter before the legislature is immaterial. Therefore, the councilman will not be liable for his statement. Note that this differs from the qualified privilege for participants in legal proceedings, which only relates to matters reasonably related to the proceeding.

respondeat superior does not apply to independent contractors. Def would be liable only if duty was nondelegable.

The correct answer is:The homeowner is liable for the maintenance company's negligence, because this is a nondelegable duty. Discussion of correct answer:Although generally persons are not liable for the acts of independent contractors that they have contracted, there are a few exceptions. One of such exceptions is when a contractor undertakes a duty the law does not permit to be delegated to another (a nondelegable duty). Examples include keeping streets in good repair, maintaining a fence around an excavation, etc. In this case, the homeowner is liable because he had a duty to use reasonable care to protect passersby on a public street from being injured by artificial conditions on his land. As such a duty is a nondelegable one, he will still be liable even though it was the independent contractor who caused the injury. Discussion of incorrect answers: Incorrect. The homeowner is liable on the basis of respondeat superior. The maintenance company is an independent contractor, not the homeowner's employee. Respondeat superior refers to the vicarious liability of an employer for the acts of his employees. As such, it does not apply in this case.

Intent is required for trespass. Accidentally hitting ball over there shows no intent.

The correct answer is:The husband is not liable for trespass, because he did not intend to cause the tennis ball to enter the neighbor's yard. Discussion of correct answer:The tort of trespass to land is defined as an intentional physical invasion of the plaintiff's possessory interest in land caused by the defendant. The defendant does not have to enter onto the land; a defendant may be liable for trespass if he causes a physical object to go onto the land. In addition, no actual injury to the land is required. However, intent is required. Under the facts presented, the wife and the husband are not liable for trespass, because they did not intend to cause the tennis ball to enter the neighbor's yard. Thus, this is the best answer. Discussion of incorrect answers: Incorrect. The husband is liable for the physical invasion of the neighbor's land by his tennis ball. The wife and the husband will not incur liability merely because their tennis ball physically invaded the neighbor's yard. The tort of trespass to land is defined as an intentional physical invasion of the plaintiff's possessory interest in land caused by the defendant. The defendant does not have to enter onto the land; a defendant may be liable for trespass if he causes a physical object to go onto the land. In addition, no actual injury to the land is required. However, intent is required, and the facts do not indicate that the wife and the husband intended for their tennis ball to enter the neighbor's yard.

If the store sells defective product it is in the market for selling, liable. Manufacturer makes a defective product, liable.

The correct answer is:The man will prevail against both the store and the manufacturer, regardless of whether the manufacturer broke the seat or the seat came that way from the components company. Discussion of correct answer:A person who assembles component parts into a finished product is strictly liable for defects in the components used. Further, one who sells any product in a defective condition unreasonably dangerous to the consumer is subject to liability for physical harm if the seller is in the business of selling such a product, and it is expected that no change will occur in the product by the time it reaches the end user. Here, the bike seat was sold in an unreasonably dangerous condition in that the seat should have been able to support a person weighing more than 150 pounds, and a reasonable purchaser would not have expected the seat to present the danger that resulted in the man's injury. Thus, it is irrelevant whether the manufacturer exercised due care; the manufacturer is liable on a strict products liability theory for the defective bicycle. Incorrect. The man will not prevail against the store, but he should prevail against the manufacturer, because they manufactured the bicycle. The store and manufacturer are both liable on a strict products liability theory. One who sells any product in a defective condition unreasonably dangerous to the consumer is subject to liability for physical harm if the seller is in the business of selling such a product and it is expected that no change will occur in the product by the time it reaches the end user. Thus, the store is subject to strict liability for defects in new goods sold by the company. With regard to the manufacturer, a manufacturer who assembles component parts into a finished product is strictly liable for defects in the components used. Here, the bicycle was clearly defective in that a bike seat should be able to support an individual weighing more than 150 pounds, and a reasonable purchaser would not have expected the seat to present the danger that resulted in the man's injury. Thus, the man should prevail against both the store and the manufacturer, and so this answer choice is incorrect.

To prevail in a public nuisance action, Pl must prove that he suffers a peculiar harm that differs from the harm suffered by the general public. In which case, he must prove special damage.

The correct answer is:There was no claim for special damages. Discussion of correct answer:A public nuisance is an act by the defendant that obstructs, inconveniences, or damages the enjoyment or use of common property. A private plaintiff may only maintain an action if he suffers an injury peculiar to that suffered by the general public. For the neighbor to prevail in a public nuisance action, he must prove that he suffers a peculiar harm that differs from the harm suffered by the general public. In which case, the neighbor must prove special damages to support his claim. Incorrect. Public use outweighs private necessity.

consumer expectation test usually deal with design defects vs manufacturing flaws which usually deal with single instances of nonconformity.

The correct answer is:This product contained a manufacturing flaw. Discussion of correct answer:This is the best answer choice available. A product manufactured in a form other than way the manufacturer intended contains a manufacturing flaw. Here, the paint flaked off as a result of an unexpected impurity in the mixture, something which would have affected only a single batch. The corporation therefore did not intend the paint to flake off in such a manner. Discussion of incorrect answers: Incorrect. The court will use the consumer expectation test standard. The consumer expectation test standard is used by courts in products liability cases involving products with design defects. Under the consumer expectation test standard, a product is in a defective condition unreasonably dangerous when it is more dangerous than would be contemplated by the ordinary consumer who purchases it, with the ordinary knowledge common to the community as to its characteristics. However, this case involves a product with a manufacturing flaw, not a design defect. Therefore, this standard is inapplicable.

Strict product liability, a person may recover damages when he is injured by a defective product. Can be manufacturer, distributor or retailer (provided it's proved sold defective).

The correct answer is:Yes, because a retailer is a proper defendant in a strict products liability suit. Discussion of correct answer:Under a theory of strict product liability, a person may recover damages when he is injured by a defective product. Liability is imposed without regard to negligence. The plaintiff must prove the product was sold in a defective condition and was unreasonably dangerous for its intended use. Three types of product defects can support a strict product liability claim. A product can be defective because of a manufacturing defect, a design defect, or because the product was not accompanied with adequate warnings to the consumer about its safe use. As in any other type of tort suit, you must name a proper defendant. Manufacturers, distributors, retailers, commercial lessors, and new home developers are potential proper defendants. Thus, the fact the retailer did not manufacture or design the product does not preclude it from being a proper defendant. Incorrect. No, because a retailer is not a proper defendant in a strict products liability suit. This answer is incorrect because it misstates the applicable legal rule. Strict liability can potentially be imposed on any commercial supplier in the distribution chain. Thus, a retailer could be subject to potential liability, as could a manufacturer. Retailers and commercial lessors are subject to strict liability for defects in new products they sell or lease. Commercial lessors can be subject to strict liability for defects in used goods that are leased. There is a split in authority among jurisdictions on whether a retailer can be subject to strict liability for defective used goods.

If not disclosed, assumed private. Publicizing such, if deemed offensive, would be invasion of privacy.

The correct answer is:Yes, because the family did not disclose their situation to anyone and it embarrassed them. Discussion of correct answer:One who publicizes a matter concerning the private life of another is subject to liability for invasion of his privacy if the matter publicized is of a kind that would be highly offensive to a reasonable person and is not of legitimate public concern. Liability may attach even if the statement or publication is true. The family must show that private facts of their life not open to public view were disseminated to the public without their permission and that the disclosure would be highly offensive to a reasonable person. Thus, this is the best answer. Incorrect. Yes, because the information could not be found in public records. It is true that, if the information could be found in public records, it cannot bring liability for public disclosure of private facts. However, the reverse is not true--if the information is not in public records, disclosure still may not result in liability. Therefore, this is not the best answer.

Can be liable for battery if what you did was substantially certain to result in contact. An assault (dog growl) can turn into batter through doctrine of transferred intent (dog bite).

The correct answer is:Yes, because the farmer intended for the dog to frighten the wife. Discussion of correct answer:To be liable for battery, a defendant must act with the intent to cause an immediate harmful or offensive contact, or know that such a contact is substantially certain to occur. Here, the farmer acted to cause the wife immediate apprehension of an offensive contact, which is the intent required for assault, not battery. However, under the doctrine of transferred intent, if a defendant acts with the intent to commit one tort against a victim and ends up committing a different tort against that victim, the intent from the first tort will "transfer" to the second tort, which will be sufficient for liability. The doctrine of transferred intent applies to assault, battery, false imprisonment, and trespass to land or chattels. Thus, in this question, although the farmer intended only to commit an assault, his intent "transfers" to the tort of battery, which is the tort that he ultimately committed when his dog bit the wife. Thus, the intent requirement is satisfied, and the farmer is liable for battery. Incorrect. Yes, because the breaking of the leash establishes liability under the doctrine of res ipsa loquitur. Res ipsa loquitur is a negligence concept, and is not applicable in assault or battery cases.

Cause-in-fact of the plaintiff's injury if that conduct was a substantial factor in causing the injury. So you knock a guy over with your car and another car hits you then you run him over, then your negligence was significant.

The correct answer is:Yes, because the man's negligence made a significant contribution to the pedestrian's injury. Discussion of correct answer:A defendant's conduct is the cause-in-fact of the plaintiff's injury if that conduct was a substantial factor in causing the injury. Each driver made a significant contribution to the injury suffered by the pedestrian. The man's significant contribution was that he knocked the pedestrian down on the ground. When there are multiple negligent parties, each of whom contributed to the plaintiff's indivisible harm, they will be jointly and severally liable. Because the harm caused here indivisible, both the man and the driver are jointly and severally liable for the pedestrian's injuries. Incorrect. No, because the other driver's act was a superseding force. The other driver's act is not considered superseding. To be superseding it must be shown that the action was unforeseeable. However, intervening negligence is generally deemed to be foreseeable.

Feeding a known vegan meat products without his knowledge is battery.

The correct answer is:Yes, if the waiter knew that the man was a vegetarian. Discussion of correct answer:The tort of battery occurs when one acts intending to cause a harmful or offensive contact with the plaintiff or with something closely connected to the plaintiff, and such harmful or offensive contact actually occurs. Causation can be either direct (e.g., direct contact with the other person) or indirect (conduct reasonably calculated to set in motion the harm-producing force). The offensiveness of the act is determined by applying a reasonable-person standard. In this case, the causation would be indirect, because the waiter tampered with the vegetarian's food with the reasonable expectation that the vegetarian would eat the food. Because the waiter's awareness of the patron's vegetarianism would make the waiter's conduct highly offensive to a reasonable person, this is the best answer. Incorrect. No, because the waiter did not come into contact with the vegetarian's body. The tort of battery does not require actual contact with the body of another person; it is sufficient if one acts intending to cause a harmful or offensive contact, and causation can be either direct (e.g., direct contact with the other person) or indirect (conduct reasonably calculated to set in motion the harm-producing force). In this case, the causation would be indirect, because the waiter tampered with the food with the reasonable expectation that the vegetarian would eat the food. Thus, this answer is incorrect.

If a store fails to notice a dangerous condition and a customer exacerbates it into a dangerous condition without telling anyone, both can be held liable.

The manager of a hardware store failed to notice that the post supporting one of the many small lighting fixtures in the hardware store's parking lot had been weakened by age and rust. Several store employees had noticed the problem over the past few months but failed to report it.A handyman, who was preoccupied with his children in the back seat, was not looking where he was going as he was backing his car into a parking space. He hit the weakened post. The post gave way slightly, but the handyman found that he could push the post back so that it looked alright, even though he knew it might give way at any minute. One week later, the post fell and struck a shopper's car as she was entering the lot to shop at the hardware store. The post fell due to the combination of its aged, rusty condition, and the force of the impact of the handyman's car. Either factor alone would not have been sufficient to cause the post to topple.The shopper can recover for the damage to her car in a negligence claim against whom? The correct answer is:The handyman and/or the hardware store, for the full amount of her damages. Discussion of correct answer:This is a negligence question. Where two or more defendants, by their concurrent negligent acts, bring about harm to the plaintiff, and it is not possible to separate portions of the harm as being the result of each act, these defendants may be held jointly and severally liable for the entire harm. The damages may be distributed between defendants who are jointly and severally liable for purposes of contribution in a separate action, or by way of impleader among the defendants themselves. However, as to the plaintiff, each defendant is liable to the plaintiff for the full amount. In this case, it is not possible to separate the portions of the harm caused by the handyman and the hardware store, so they will be held jointly and severally liable for the entire harm. Incorrect. The hardware store, but not the handyman. The hardware store and the handyman will be held jointly and severally liable for the plaintiff's injury, because it is impossible to separate the portions of the harm caused by their actions.

If an employee's actions were not committed to further the employer's business objectives, then the employer will bear no liability for the employee's actions. The "scope of employment" includes acts so closely connected and reasonably incidental to what the servant was employed to do that they may be regarded as methods, however improper, to carry out the employer's objectives

Two brothers decided to open a dry cleaning business. After the brothers had been running the business for several months, they realized that although the volume of business exceeded their expectations, they were barely breaking even. Investigating further, the brothers determined that the cash registers and bank accounts did not contain the expected amount of funds. They installed a video camera in the shop and quickly discovered that an employee was stealing money. The video also included footage of the employee physically threatening a customer and stealing his clothes. The brothers immediately fired the employee.If the customer the ex-employee stole from and threatened sues the brothers, what is the likely outcome of the customer's lawsuit? The correct answer is:The brothers will bear no liability for the ex-employee's acts, because the ex-employee did not commit the acts against the customer for the benefit of the brothers. Discussion of correct answer:An employer is liable for the tortious acts of his or her employees that are committed within the scope of employment and that cause injuries or property damage to a third person. The "scope of employment" includes acts so closely connected and reasonably incidental to what the servant was employed to do that they may be regarded as methods, however improper, to carry out the employer's objectives. Here, the ex-employee's actions amount to an intentional tort. However, because the actions were not committed to further the brothers' business objectives, the brothers will bear no liability for the ex-employee's actions. Discussion of incorrect answers: Incorrect. The brothers will be liable only for the customer's clothing, because there is no way to prove how much money the ex-employee stole from the customer. This is not the correct answer, because the mere fact that damages may be difficult to prove will not relieve an employer of liability he would otherwise bear for the acts of his former employee. An employer is liable for the tortious acts of his or her employees that are committed within the scope of employment and that cause injuries or property damage to a third person. The "scope of employment" includes acts so closely connected and reasonably incidental to what the servant was employed to do that they may be regarded as methods, however improper, to carry out the employer's objectives. Thus, the answer to the question hinges on whether the ex-employee's tortious acts were committed within the scope of his employment.

Where a defendant acts with the intent to inflict a tortious injury on a person, but for some reason causes injury to a different victim than intended, the defendant's intent is considered "transferred" to the actual victim. The reason for the motive in the hit is less important.

Two talented figure skaters were competing for a place on the United States Olympic team. One afternoon before the Olympic trials, the older skater warned the younger skater that if she did not withdraw from the competition, the older skater would break both of the younger skater's legs. The younger skater's coach overheard this conversation and became angry. He took a swing at the older skater but missed, striking another competitor.If the other competitor brings a battery suit against the coach, will she prevail? The correct answer is:Yes, despite the fact that the coach intended to hit the older skater. Discussion of correct answer:An actor is subject to liability to another for battery if he acts intending to cause a harmful or offensive contact and an offensive contact directly or indirectly results. Where a defendant acts with the intent to inflict a tortious injury on a person, but for some reason causes injury to a different victim than intended, the defendant's intent is considered "transferred" to the actual victim. This transferred intent applies only to assault, battery, false imprisonment, and trespass (to land or to chattels). Here, the coach's intent to strike the older skater is transferred to the other competitor, and because the harmful contact actually occurred, the coach is liable for battery Incorrect. Yes, because the coach had an improper motive in swinging at the older skater. While it is true that under the theory of transferred intent, the coach is liable for striking the other competitor, the coach's motive in swinging at the older skater is irrelevant. Motive may be relevant to certain criminal offenses, but it is not an element of the intentional tort of battery; intent to cause harmful or offensive contact is sufficient. Thus, this answer is incorrect.

Attractive nuisance doctrine holds land possessor liable based on foreseeability of harm to the child. If you're aware kids play on your driveway don't leave dangerous stuff there.

Discussion of correct answer:Under the attractive nuisance doctrine, a possessor of land may be liable for physical harm to child trespassers caused by an artificial condition on the land if (1) the possessor knows or has reason to know that children are likely to trespass; (2) the possessor knows or should know that the condition poses an unreasonable risk of serious harm to children; (3) the children, due to their age, would not discover the condition or realize the risk it involves; (4) the possessor's need to maintain the condition and the burden of eliminating the danger are slight compared to the risk to children; and (5) the possessor does not exercise reasonable care to eliminate the danger or otherwise protect the children. This doctrine is based on foreseeability of harm to the child, not attraction onto the property, and the artificial condition must pose an unreasonable risk of harm in light of the particular child's age. Here, the neighbor owed the child the duty of reasonable care to prevent the accident. Given that the neighbor was aware that the child often played in the neighbor's driveway, it was foreseeable that the child would trespass and that, given his age, the child might be attracted to machinery in the neighbor's yard. A court is likely to find the neighbor liable for the child's injuries. Discussion of incorrect answers: Incorrect. The neighbor, because he had no duty to maintain or inspect the equipment. This answer is incorrect because it makes a true statement but reaches a false conclusion. The neighbor had no duty to inspect to inspect the equipment, but he did have a duty to exercise reasonable care to prevent harm to the child. Under the attractive nuisance doctrine, a possessor of land may be liable for physical harm to child trespassers caused by an artificial condition on the land if (1) the possessor knows or has reason to know that children are likely to trespass; (2) the possessor knows or should know that the condition poses an unreasonable risk of serious harm to children; (3) the children, due to their age, would not discover the condition or realize the risk it involves; (4) the possessor's need to maintain the condition and the burden of eliminating the danger are slight compared to the risk to children; and (5) the possessor does not exercise reasonable care to eliminate the danger or otherwise protect the children. The neighbor knew that the child often played in his driveway, and that the child was so young that he was unable to accurately assess the risk on playing on heavy equipment. The neighbor owed the child the duty of reasonable care to prevent the accident.

If it wasn't intentional, it would have to be negligence. If no evidence of unreasonable actions then no liability.

The correct answer is: Grant the motion, because there is no evidence of negligence by the woman. Discussion of correct answer:Because this was not an intentional tort, the only claim that the man can bring is a negligence claim. In such a claim, the plaintiff must prove that the defendant did not act reasonably and hence breached a duty to the plaintiff causing damages. In this case, the only evidence offered at trial is that there is a defect in the carburetor of the woman's automobile. There is no evidence that the woman knew of the defect or hired an incompetent mechanic to repair the vehicle. There is no evidence that she acted unreasonably. Therefore, there is no basis for the woman to be liable. Incorrect. Deny the motion, because the man was injured by a defect in the automobile. There is no basis upon which to hold the woman strictly liable for driving a car. The only evidence introduced points to potential liability by the manufacturer.

Respondeat superior vs Contributory negligence. Respondeat superior is a total defense, whereas contributory negligence is not.

The correct answer is: Respondeat superior. Discussion of correct answer:Under the doctrine of respondeat superior, an employer is liable for injuries caused by the negligence or strict liability of an employee, if the tortious act occurred within the scope of the employment. Acts are within the scope of employment if they are so closely connected with what the employee was hired to do and so fairly and reasonably incidental to it that they may be regarded as methods, even though improper, of carrying out the objectives of the employment. If an employee uses force, even misguidedly, wholly or partly to further the employer's purpose, such use of force may fall within the scope of employment, resulting in vicarious liability for the employer. However, employers are generally not liable for the intentional torts of their employees. Here, although the watchman was on duty at the time of his assault on the addict, the addict was not threatening the employer or the employer's property. The watchman used force in response to the addict's verbal taunts and was not furthering the employer's purpose in any way when he did so, because the taunts were directed at the watchman personally and were not directed at the manufacturer. Under the theory of respondeat superior, the manufacturer should not be liable for the addict's injuries. Incorrect. Contributory negligence. This is not the correct answer because it is not the best theory the manufacturer can raise in its defense. The theory of contributory negligence requires a plaintiff to exercise due care to protect himself from injury by a defendant, and to behave reasonably under the circumstances. However, if the plaintiff's theory is intentional tort, recklessness, or strict liability, the plaintiff's contributory negligence does not bar recovery from the defendant. Rather, the plaintiff's recovery may be adjusted to account for his own negligence. Thus, although the manufacturer may want to assert, among other arguments, that the addict was negligent in harassing the watchman, contributory negligence is not the best defense the manufacturer can present.

transferred intent doctrine: You hit someone's stick, it goes flying, nearly thwacks someone and causes them to fall. Battery. Even if you were fencing with the first stick owner. Also ties into negligent conduct. Double whammy.

The correct answer is: Yes, for negligence and battery, because of the transferred intent doctrine. Discussion of correct answer:A battery is an intentional act that causes a harmful or offensive contact with the plaintiff or with something closely connected thereto. If the hiker desired or knew to a substantial certainty that he would cause a harmful or offensive touching to the other hiker by striking him with the walking stick, that intent will transfer to provide the intent element of a cause of action for battery for the wife against the hiker. The hiker put the force in motion that ultimately led to the harmful touching (the wife falling to the ledge). It is important to note that the fact that the other hiker consented to the hiker's playfully hitting him with the stick will not operate as a defense, because consent as a defense is personal to the person consenting. With this defense, the person committing the act is still committing a tort (in this case, battery). However, because the plaintiff consented to the action, they cannot then claim the tort against the defendant. With the transferred intent doctrine, the intent of the tort being committed is transferring from being against the intended victim to being against the actual victim. This means that defenses personal to them, like consent, are not going to apply, and would not transfer. The defendant is still committing a tort, just against someone else. Discussion of incorrect answers: Incorrect. Yes, for negligence only, because a reasonable hiker would not risk playing with sticks that can fall on hikers below him. The hiker might also be liable for battery, and not just for negligence.

In the majority of jurisdictions, the measure of damages for misrepresentation is measured by the benefit of the bargain theory. What you thought it was worth minus what it was actually worth.

The correct answer is:$100,000. Discussion of correct answer:The businessman will be liable for misrepresentation because of his active concealment of the true condition of the porches. In the majority of jurisdictions, the measure of damages for misrepresentation is measured by the benefit of the bargain theory. The historian is entitled to damages measured by the difference between the value of the property he was buying if its condition was as represented and its value with defects. In this instance, the historian bought property that was worth $500,000 in good condition, but was worth $400,000 in its true condition. He is entitled to $100,000 in damages. Incorrect. $40,000. Under the majority rule, the historian is entitled to his benefit of the bargain ($100,000). The minority rule is the out-of-pocket test--the difference between what the plaintiff paid and the value of what he received ($60,000). The amount of $40,000 is not the proper amount of damages under either test.

Private necessity as applied to trespass to chattels doesn't require replacement of entire worth (break and buy) just it's diminution in value.

The correct answer is:$8,000. Discussion of correct answer:This is a tricky Torts question dealing with the defense of private necessity. As a general rule, private necessity will excuse tortious conduct aimed at property. However, according to Restatement of Torts 2d, Section 263, which specifically addresses the privilege of private necessity as applied to trespass to chattels or conversion, "Where the act is for the benefit of the actor or a third person, he is subject to liability for any harm caused by the exercise of the privilege." Therefore, the man would be liable for the damage caused to the owner's boat. Note that choice (D) is incorrect because the man is not liable for conversion, as he only destroyed the mast of the boat and not the whole thing. Incorrect. $25,000.

Private v Private Defamation Claim: slander per se does not require a showing of actual pecuniary loss as a precursor to any recovery. So both actual and presumed damages can be gained.

The correct answer is:Actual and presumed damages. Discussion of correct answer:When a private plaintiff sues a private defendant for defamation regarding a matter of purely private concern, the plaintiff need only show negligence with regard to the truth or falsity of the statement. Here, the tailor and the bank representative are both private persons, and the tailor's loan history does not appear to be information that has an impact on the public good. Although the statement is slander, it reflects on the tailor's conduct of his business and thus is probably slander per se, which does not require a showing of actual pecuniary loss as a precursor to any recovery. The bank representative made a false statement without checking information, which is probably readily available to him, so he would probably be found negligent. Therefore, the tailor could recover both actual and presumed damages. Incorrect. Any actual damages. The bank representative's statement reflects on the tailor's conduct of his business and thus is probably slander per se, which does not require a showing of actual pecuniary loss as a precursor to any recovery. Therefore, the tailor could obtain both actual and presumed damages. Thus, this answer is incorrect.

Commercial suppliers at all levels of distribution (making, distributing, selling) may be sued. Even if not strictly liable, sellers can be sued for negligence.

The correct answer is:Against the manufacturer, the distributor, and the retailer only. Discussion of correct answer:In a suit for strict products liability, commercial suppliers at all levels of the distribution chain (i.e., manufacturer, distributor, retailer) are all potential defendants. Therefore, it is true that, in this case, the manufacturer, distributor, and retailer of the toothbrush may be sued by the injured woman. Occasional sellers and those supplying services (such as the dentist in this case) cannot be strictly liable, but may be sued for negligence. Incorrect. Against the manufacturer only. Even if the manufacturer were responsible for the woman's injuries, the manufacturer will not be the only potential defendant. In a suit for strict products liability, commercial suppliers at all levels of the distribution chain (i.e., manufacturer, distributor, retailer) are all potential defendants. This is true regardless of whether the party is actually at fault for the injury.

Battery doesn't require hitting the person, it can be hitting an object held by the person.

The correct answer is:Battery, because the referee was holding the puck. Discussion of correct answer:A battery is an intentional act that causes a harmful or offensive contact with the plaintiff or any item connected to the plaintiff. An actual touching of the person is not required. In this case, the player's intent to touch the puck that the referee was holding in his hands is sufficient to be a battery. Incorrect. Battery, but only if the player hit the referee's hand in addition to the hockey puck. A battery requires an offensive touching of another, but the touching need not always be a touching of the plaintiff's body. It can be a touching of an object connected to the plaintiff's body. Because the hockey puck was in the referee's hand when it was batted away, this was a touching for battery purposes. Thus, this answer is incorrect.

comparative negligence regime, where the plaintiff's negligence has contributed to her own injuries, the total damages caused by the defendant may be apportioned based upon a determination of the relative fault of each party

The correct answer is:Because she sued in a pure comparative negligence jurisdiction, and the jury found her 90% responsible and found the bookstore owner only 10% responsible. Discussion of correct answer:The second customer would be able to recover for the portion of her damages for which the bookstore owner was considered responsible (10%). Under a comparative negligence regime, where the plaintiff's negligence has contributed to her own injuries, the total damages caused by the defendant may be apportioned based upon a determination of the relative fault of each party. A majority of states have adopted comparative negligence systems, either by statute or judicial decision. Some states have adopted a "pure" system of comparative negligence. In such states, apportionment of damages tracks apportionment of fault perfectly—if the defendant is 25% responsible and the plaintiff is 75% responsible, the plaintiff recovers from the defendant 25% of the total damages she suffered. Incorrect. Because she sued in a contributory negligence jurisdiction, and the jury found her less responsible than the bookstore owner. In a contributory negligence jurisdiction any negligence of the plaintiff bars the plaintiff from recovery. Contributory negligence requires the plaintiff to exercise due care to protect herself from injury by the defendant. The analysis is similar to that for the defendant's negligence: did the plaintiff act as a reasonable person would under the same circumstances? If the plaintiff is found to have engaged in contributory negligence, she is barred from recovery for the defendant's otherwise negligent conduct. Consequently, even though the jury found the second customer less responsible than the bookstore owner, she would be barred from recovery in a contributory negligence jurisdiction.

Truth, even if just believed, is a defense against defamation claims. There is no hearsay loop when it comes to defamation.

The correct answer is:Brent was responding to a request for a job recommendation about Arnold and was explaining why Brent's company had to let Arnold go. Arnold did not get the job as a result of the statement. Discussion of correct answer:If the speaker believes the statement to be true and is communicating a matter of interest, he has qualified immunity from defamation. One example of this is in requests for job references. Incorrect. Brent was merely repeating what Jason had told him, and Brent made clear that he was only repeating something he had heard. Any person who repeats the defamatory message is liable as a publisher. This is so even where the repetition is qualified by such terms as "alleged" or is said not to be the opinion of the republisher. Therefore, the fact that Brent made clear that he was only repeating what Jason told him does not weaken Arnold's case. Thus, this answer is incorrect.

False Imprisonment requires intent to imprison.

The correct answer is:For the store, because the store employees did not know the late shopper was in the store when they closed. Discussion of correct answer:A person is subject to liability to another for false imprisonment if: (1) he acts intending to confine a person within certain fixed boundaries; (2) his act directly or indirectly results in such a confinement of the other; and (3) the other is conscious of the confinement or is harmed by it. Here, the late shopper knew that the store was due to close, but he continued to shop. The store locked up after the salespeople closed down their departments, and no one saw that the late shopper was still in the store. Given that there was no intent to confine the late shopper, he will not prevail. Incorrect. For the late shopper, because the store employees had a responsibility to find him before locking up. A person is subject to liability to another for false imprisonment if: (1) he acts intending to confine a person within certain fixed boundaries; (2) his act directly or indirectly results in such a confinement of the other; and (3) the other is conscious of the confinement or is harmed by it. This answer incorrectly implies that one can be liable for false imprisonment with a mens rea of recklessness or negligence. In fact, an actual intent to confine is required.

The measure of damages for a trespass action is the actual amount of loss suffered by reason of the impairment of the property or the loss of its use. No damage, no claim.

The correct answer is:Grant judgment as a matter of law for the striker, because the employee presented no evidence of damage to his automobile. Discussion of correct answer:The action of trespass to chattels involves an intentional interference with the plaintiff's possessory interest in personal property. It is the proper action where the defendant's interference does not amount to a substantial interference with the plaintiff's possession of the property (i.e., an exercise of dominion and control over the property) and only consists of inter-meddling with, use of, or damage to the plaintiff's personal property. The measure of damages for a trespass action is the actual amount of loss suffered by reason of the impairment of the property or the loss of its use. The judge should grant judgment as a matter of law for the striker unless the employee can establish a claim for trespass to chattels. One essential element of the plaintiff's case for trespass to chattels is actual harm. Because the striker did not deprive him of possession, the employee must show that the striker damaged the car. The judge can only grant judgment as a matter of law for the employee if he proves all the elements of an action for trespass to chattels. This includes proof of actual harm, which is required to assert an action for trespass to chattels (unlike a case of trespass to real property, where no proof of actual damage is required). Thus, the judge cannot direct a verdict for the plaintiff, even on the issue of liability, until the plaintiff has provided proof of actual damage. Discussion of incorrect answers: Incorrect. Grant judgment as a matter of law for the employee on the issue of liability. The judge cannot direct a verdict for the plaintiff, even on the issue of liability, until the plaintiff has provided proof of actual damage.

Where expert knowledge isn't needed to tell if a professional messed up then the judgement can be made from the juror's experience (provided judge believes jury is competent).

The correct answer is:If the judge agrees that the jurors, based on their common knowledge, can determine if the lawyer acted as a reasonably prudent lawyer in good standing. Discussion of correct answer:Whether a defendant has breached his duty to the plaintiff is a question of fact to be determined by the jury. In most professional malpractice cases, the jury receives expert testimony as to whether the defendant has failed to act in accordance with general standards of practitioners in good standing. However, in some instances no special knowledge is necessary to determine that a professional has performed inadequately. That judgment can be made from the jurors' experience. If the judge determines that the jury is competent enough to judge the reasonableness of a failure to serve a lawsuit, then no expert testimony will be necessary to prove breach. Incorrect. Only if an attorney expert from the same community testifies that the lawyer did not act as a reasonably prudent lawyer in good standing. The modern rule is that the expert does not have to come from the same community. Professional education is now largely nationally accredited, so the training of professionals does not vary dramatically from place to place as it used to. Also, expert testimony is not the only way the woman might be able to establish a breach of the standard of care. If the professional's conduct is such that a lay person is capable of judging it, expert testimony will not be required.

Joint and several liability applies unless told otherwise. This is designed to insure that the plaintiff receives full compensation. Defendants can fight over what each other owes themselves.

The correct answer is:Joint and several liability should be imposed on the cyclist and the rollerblader for the full amount of damages. Discussion of correct answer:The examiners state that you are to assume joint and several liability applies unless told otherwise. Therefore, the cyclist and the rollerblader are jointly and severally liable for the man's injuries. This is designed to insure that the plaintiff receives full compensation. The cyclist and rollerblader can determine their relative liabilities between themselves, but that burden and risk of partial compensation does not lie on the plaintiff. Discussion of incorrect answers: Incorrect. Liability should be divided between the cyclist and rollerblader in proportion to their negligence. A division of liability according to fault will not be reflected in the man's judgment, even in most comparative negligence jurisdictions. That would be determined in a separate lawsuit between the cyclist and the rollerblader.

Where children are known trespassers, reasonable care is expected. Defendant must exercise due care.

The correct answer is:Judgment for the boy, if the company knew that children crossed the tracks and could readily have prevented this. Discussion of correct answer:The company may have used reasonable care to stop theft of the "third rail" insulation and still be liable in negligence if options were available to them to prevent known child trespassers from coming in contact with an exposed power rail, and those options were less burdensome than the injuries threatened, which were probable and grave. Even though the company took reasonable steps to deal with insulation thefts, it knew that they presented an ongoing risk to anyone on the tracks near such a theft, and if it also knew that children were frequent trespassers in the area of the park, that the children would not recognize the danger, and it could easily prevent such trespassing, it would have failed to exercise due care as to the children. Note also that the boy only needed to climb a low fence to get to the tracks. Incorrect. Judgment for the company, because the intentional criminal act of stealing "third rail" insulation was an intervening, superseding cause of the boy's injuries. Even though the theft of the insulation was an intentional criminal act, such acts will not be superseding of the company's negligence as to the children because the intentional acts were highly foreseeable and because the company's negligence in not preventing trespassing increased the risk to the children from the power rail being so exposed.

Landowner only owes duty of care to foreseeable child trespassers as to conditions he KNOWS about. If he didn't know about the condition, then no duty to unknown trespasser.

The correct answer is:Judgment for the businessman, because he was not actually aware of the rusted hobby horse support, and the boy was a trespasser. Discussion of correct answer:According to the attractive nuisance doctrine, a landowner owes a duty to foresee-ably trespassing children when he knows or reasonably should know of an unreasonably dangerous artificial condition on his property. The landlord's liability arises when he has knowledge or is put on notice of the dangerous condition. He has no duty to inspect his land to find conditions of which he is unaware. Since the boy is a child trespasser, the businessman was unaware of the rusted hobby horse support, and there are no facts from which the businessman should have realized the condition of the rusted spring, the businessman is not liable to the boy for the injuries suffered. Incorrect. Judgment for the boy, because a landowner must exercise due care to inspect for and discover concealed artificial conditions which threaten known trespassers. This is an incorrect statement of law. A landowner must exercise due care regarding foreseeable child trespassers as to dangerous artificial conditions of which he knows or should reasonably know - those conditions of which he is put on notice by facts that would lead a reasonable person to discover the condition. There is no affirmative duty to inspect the premises (absent notice) to discover any concealed dangerous conditions.

A defendant owes child trespasser reasonable care where it concerns artificial conditions and the burden of safety is less than the risk to the child.

The correct answer is:Judgment for the plaintiff, because the child did not appreciate the dangers of climbing on the storage tank. Discussion of correct answer:A land occupier owes a duty of reasonable care to a child trespasser when all of the following conditions are met: (1) the child is encountering an artificial condition; (2) the land occupier knows or has reason to know that children are likely to trespass on his property; (3) the land occupier knows or has reason to know that there are conditions on his property that are unreasonably dangerous to children; (4) the children because of their immaturity do not recognize and appreciate the danger posed by the condition; and (5) the burden to correct the condition is substantially less than the risk to the trespassing children. The storage tanks are an artificial condition. The defendant's agent knows that children are likely to trespass on the land because there are many children in the surrounding residential neighborhood and because he has recently discovered a child's jacket and lunch pail on the premises. A fall from the ladder either into the tank or onto the ground could be fatal to a child. Moreover, an eight-year-old is not likely to understand how serious the risks could be. The burden of carrying out repairs to the fence immediately, or even of arranging for enhanced security presence, such as guards or dogs, is small compared to the danger the artificial condition poses to neighborhood children. Therefore, the defendant owed the child a duty of reasonable care. Incorrect. Judgment for the defendant, because the injury would not have occurred but for the plaintiff's intentionally tortious act of vandalism. This answer is incorrect. First, the facts do not indicate that the injured child cut the fence; someone else may have committed the vandalism and the child merely used the opening in the fence in order to gain access to the property. Second, even if the child did cut the fence, the fence was cut before Saturday, and the injury did not occur until Tuesday at the earliest. The defendant had ample time to repair the damage and prevent the child from gaining access to the property.

Intentional Infliction of emotional distress & possible transferred intent requires full awareness or at least it's foreseeable.

The correct answer is:Judgment for the waiter, because he was unaware of the mother's presence in the restaurant. Discussion of correct answer:A defendant may be liable for intentional infliction of emotional distress when he engages in extreme and outrageous conduct toward one person, intending to inflict emotional distress upon a family member of the victim whom the defendant knows is present. Whether viewed as a limited application of the "transferred intent" doctrine or of the principle that the defendant must be substantially certain that a family member witnessing his attack upon the victim will suffer emotional distress, the defendant must be aware of the family member's presence. Because the waiter was unaware of the mother's presence in the restaurant, he cannot have had the requisite intent as to her. Incorrect. Judgment for the mother, if the waiter's conduct was extreme and outrageous. The waiter's conduct being extreme and outrageous alone is insufficient to prove intentional infliction of emotional distress to a bystander. The waiter must have intended that the mother, the bystander, suffer emotional distress. If the waiter did not know that the mother was present, he cannot have had such an intent. Thus, this answer is incorrect.

Attractive Nuisance does not apply if the child had repeat warnings such as posted warning signs.

The correct answer is:No, because the 12-year-old boy was warned of the danger. Discussion of correct answer:Because the 12-year-old boy saw the signs and was aware of the risk, he will be treated as an adult trespasser, and so the attractive nuisance doctrine will not apply. As such, the boy assumed the risk of injury when he ignored the posted warning signs. Incorrect. Yes, because the construction company created a public nuisance. This answer is incorrect. A public nuisance is an unreasonable interference with a right common to the general public. In determining whether an interference with a public right is unreasonable, courts will consider the following factors: 1) whether the conduct involves a significant interference with the public health, the public safety, the public peace, the public comfort, or the public convenience; 2) whether the conduct is proscribed by a statute, ordinance, or administrative regulation; or 3) whether the conduct is of a continuing nature or has produced a permanent or long-lasting effect, and, as the actor knows or has reason to know, has a significant effect upon the public right. Under this analysis, the construction project did not create a public nuisance.

"Do not park here" is not a basis for recovering if the truck explodes from an unforeseeable event and the driver not negligent in leaving it there & reasonably entitled to park there.

The correct answer is:Judgment for the young man, because his parking his truck next to the homeowner's home was not a negligent act. Discussion of correct answer:When a defendant violates a criminal statute, that violation may be considered negligence per se (majority rule) or may raise a rebuttable inference of negligence (one minority rule) if all of the following conditions are met: (1) the injury caused by the defendant's violation is the type which the statute was intended to prevent; (2) the plaintiff is a member of the class intended to be protected; and (3) the defendant's violation is not excused. The injury sought to be prevented by a statute prohibiting vehicles over a certain weight on certain roads is the damage that such heavy vehicles do to the roadway. The purpose of the statute is not to protect against the type of injury that occurs in this case. It was not negligent for the young man to park his truck next to the homeowner's house, since the young man is reasonably entitled to rely upon the fact that other drivers on the road will operate their cars prudently under normal circumstances. Parking in that particular spot did not foresee-ably increase the risk that a collision would occur, and thus was not negligent. Because the young man was not negligent, the homeowner has no basis upon which to recover from him. Incorrect. Judgment for the homeowner, because the young man's parking his truck next to her home was a contributing cause of her injury. The presence of the young man's truck contributed in fact to the destruction of the homeowner's home, but because the young man was not negligent in parking there, there is no basis upon which to predicate his liability.

Where harm is indivisible, defs are jointly and severally liable. Even in comparative negligence areas this is still the case - but the defs would have burden of sorting out on their own.

The correct answer is:Liability imposed upon either defendant for the full amount of the plaintiff's injury. Discussion of correct answer:The homeowner and passerby combined to cause the plaintiff's injuries. Since the harm is indivisible, they are jointly and severally liable for the full amount of his damages. In the majority of jurisdictions, even where comparative negligence has been adopted, any reallocation of the damages between the defendants will be their burden. This ensures that the plaintiff gets one full recovery. Incorrect. Liability divided between the homeowner and the passerby in proportion to their fault. Even where comparative negligence has been adopted, most jurisdictions have retained the concept of joint and several liability. Hence the defendants will be jointly and severally liable where the harm is indivisible, as in this case.

Defendant can be held liable for negligent action of an independent contractor if the activity in question was non-delegable.

The correct answer is:Negligence. Discussion of correct answer:A defendant is not liable for torts committed by someone he has engaged as an independent contractor. This is because the defendant has no right to control the activity of the contractor. In three situations, contrary to the general rule set forth above, a defendant may be held vicariously liable for the torts of an independent contractor. One of these situations is where that duty is nondelegable, in which case the person who hired the independent contractor, as well as the independent contractor, will be liable. Discussion of incorrect answers: Incorrect. Strict liability. The fact the tree was on the homeowner's property does not create strict liability. The homeowner can however be held vicariously liable for the handyman's actions.

A school teacher is not a public figure for defamation purposes. A private person suing a media defendant for defamation must show that the defendant was at least negligent with regard to the truth or falsity of the printed statement.

The correct answer is:Negligent publication. Discussion of correct answer:To prove an action for defamation, the teacher will have to show that: 1) a false, defamatory statement of fact was made; 2) reasonably understood as relating to the teacher; 3) intentionally or negligently published to a third party; 4) causing her damage; 5) made with the requisite degree of fault as to the truth or falsity of the statement. Under Gertz v. Welch [418 U.S. 323 (1974)], a private person suing a media defendant for defamation must show that the defendant was at least negligent with regard to the truth or falsity of the printed statement, where the statement involves a matter of public concern, such as the performance of public school teachers. Although the teacher was the author of several books on teaching English to the disadvantaged, she is not a public figure. She has not achieved an all-pervasive fame, attempted to use the media to gain notoriety, nor injected herself into the public arena to attempt to direct public policy. She is a local English teacher, a private person. Thus, if the newspaper or its agent, the reporter, were not negligent in investigating the truth or falsity of the statement, the teacher cannot recover in defamation. Choice (A) is incorrect as malice is too high of a standard. The teacher must prove the article was negligently published in order to recover. Choice (D) is incorrect. More than mere publication is necessary since this is a matter of public concern. C) Incorrect. Reckless publication.

A defendant is not liable for the torts committed by someone as an independent contractor. And an employer is only liable for an employee acting within his duties as an employee, not what he does on the side.

The correct answer is:Neither the truck driver's employer nor the farmer. Discussion of correct answer:A defendant is not liable for the torts committed by someone he has engaged as an independent contractor. Here, the truck driver was hired as an independent contractor to deliver Christmas trees. The farmer had no control over the truck driver's activity, other than directing him where to deliver the Christmas trees. The farmer is not vicariously liable for the truck driver's actions. Neither can the truck driver's employer be held liable for the pedestrian's injuries. An employer is liable for injuries caused by the negligence of an employee only if the tortious act occurred within the scope of the employment. In this case, the driver was not performing work for his employer when the accident occurred; he was moonlighting on his own time. Therefore, the pedestrian cannot recover from either company. Discussion of incorrect answers: Incorrect. Both the truck driver's employer and the farmer. A defendant is not liable for the torts committed by someone he has engaged as an independent contractor, so the farmer cannot be held liable for the pedestrian's injuries. As to the truck driver's employer, an employer is liable for injuries caused by the negligence of an employee only if the tortious act occurred within the scope of the employment. In this case, the truck driver was moonlighting for the farmer on his own time when the accident occurred, so the truck driver did not commit a tort within the scope of his employment.

If express consent is given, defendant would have know of and take advantage of a mistaken belief to be held liable.

The correct answer is:No, because the blue team member had given her express consent. Discussion of correct answer:Where a defendant has otherwise committed a tort, consent by the plaintiff may work as a defense. Consent is defined as the plaintiff's willingness in fact that the defendant's action take place. Consent may be granted orally or it may be implied from the plaintiff's conduct. As a regular participant in the paintball league, the blue team member clearly consented to the regular risks of the sport. Furthermore, the blue team member expressly consented to the red team captain's suggestion that they "kick things up a notch." In cases of intentional torts, express consent remains valid even when granted by mistake, unless the defendant knows of or takes advantage of that mistake. Here, the blue team member was wearing her protective gear as usual, and her injury occurred shortly after the teams resumed the game using the hard-shelled, Type B pellets; thus, there were no external indicators to the red team captain that the blue team member did not expect the shift from soft-shelled to hard-shelled pellets. As such, although the blue team member granted her consent without realizing the implications of the red team captain's question, it does not appear that the red team captain knew of or took advantage of the blue team member's mistake. The blue team member would not prevail in a battery claim, because the actions of the red team captain would be excused through a successful defense. Incorrect. No, because the blue team member had given her implied consent. In cases of intentional torts to the person, consent is defined as the plaintiff's express or implied willingness that an invasion of her interest shall take place. Here, the blue team member had played many paintball games prior to the one in question. Clearly her consent to the normal risks of the game was implied. If that were the only type of consent at issue, however, the blue team member might have a valid case that the use of the hard-shelled pellets exceeded the scope of the plaintiff's consent. But here, the blue team member gave her express consent to "kick things up a notch." Even though the blue team member made a mistake when she gave her consent, because she did not realize what she was agreeing to, for mistake to negate the defense of consent, the defendant has to have been aware of the mistake. There is no evidence that the red team captain had any knowledge of the blue team member's mistake, and so the express consent to "kick it up a notch," rather than implied consent, works to relieve the defendant of liability.

Owner of wild animal might not be liable if plaintiff assumed risk in approaching or goading.

The correct answer is:No, because the friend knowingly and unreasonably subjected himself to a risk of harm. Discussion of correct answer:An owner of a wild animal, even a wild animal kept as a pet, is strictly liable for injuries caused by a dangerous propensity common to the species of animal in question. However, the victim must do nothing voluntarily to cause the animal to injure him. The fact that the friend knew that the woman owned a tiger cub does not provide the woman with a defense. The friend did not know that the cub was not in its room. However, assumption of risk is a defense to strict liability when the plaintiff appreciates the danger and voluntarily subjects himself to the danger. Once the friend discovered that the cub was out of its room, he batted its face with the toy, causing the cub to bite him. Therefore, the friend unreasonably and voluntarily took on a known risk. Because the friend thereby assumed the risk of injury, he cannot recover from the woman for his injuries. Incorrect. Yes, because, as owner of the tiger cub, the woman is strictly liable for the friend's injuries. An owner of a wild animal, even a wild animal kept as a pet, is strictly liable for injuries caused by a dangerous propensity common to the species of animal in question. Of course, the victim must do nothing voluntarily to cause the animal to injure him. In this question, once the friend discovered that the tiger cub was out of its room, he batted it in the face, causing the cub to bite him. Therefore, the friend unreasonably and voluntarily took on a known risk. Thus, he cannot recover from the woman for his injuries.

"shopkeeper's privilege," a businessman who reasonably suspects a customer of theft or of failure to pay may detain the suspected individual for a short time in order to investigate. Key issue in such a question is whether owner or employee acted reasonably.

The correct answer is:No, because the guard acted reasonably in believing a theft had occurred. Discussion of correct answer:According to the "shopkeeper's privilege," a businessman who reasonably suspects a customer of theft or of failure to pay may detain the suspected individual for a short time in order to investigate. The privilege is, however, a very restricted one, confined to what is reasonably necessary for its limited purpose of enabling the defendant to do what is possible on the spot to discover the facts. There will be liability if the detention is for a length of time beyond that which is reasonably necessary for such a short investigation. In the present case, the security guard was privileged to make a temporary detention since he reasonably believed that a theft had occurred. As a result, choice (C) is correct. (D) is less preferred because the woman is asserting an action for false imprisonment. Thus, it is irrelevant whether her seizure was foreseeable. The key issue here is whether the guard acted reasonably under the circumstances. D) Incorrect. No, because it was unforeseeable that a temporary detention would result in the detainee suffering a diabetic seizure.

A rescuer is a foreseeable plaintiff. The rescuer would have to be wantonly reckless to let the negligent defendant off the hook.

The correct answer is:No, because the man was acting to prevent injury to the persons in the path of the vehicle. Discussion of correct answer:A negligent defendant owes a rescuer an independent duty of care. Courts recognize that a rescuer is a foreseeable plaintiff. Because the man was injured attempting to stop the car to prevent it from hitting other guests at the party, the man qualifies as a rescuer. Even when a rescuer is negligent in acting, the negligent defendant will be liable unless it is shown that the rescuer was wantonly reckless. The facts do not indicate the man acted wantonly reckless in attempting to prevent injury to others. Incorrect. Yes, the best friend did not owe the man any duty. This answer is incorrect because it misstates the applicable legal rule and reaches the wrong legal conclusion. It is true that before liability can be imposed on a defendant for negligence, a duty must be shown to exist that required the defendant to conform his conduct to a standard for the protection of the plaintiff. Generally, a negligent defendant owes a rescuer a duty of reasonable care because a rescuer is a foreseeable plaintiff. Because the man was acting as a rescuer, the best friend owed him a duty of reasonable care.

Where contributory negligence applies, decedent's negligence or the plaintiff's negligence could bar all recovery.

The correct answer is:She can collect only if neither she nor the boy were negligent. Discussion of correct answer:In jurisdictions that apply contributory negligence, either the decedent's negligence or the plaintiff's negligence could prevent a recovery in an action for wrongful death. In this question, negligence by either the mother or the boy would bar all recovery. Incorrect. She can collect if she was negligent and as long as the boy was not negligent. Because the beneficiary is the actual plaintiff in an action under the typical wrongful death statute, all the usual defenses, including contributory negligence, apply and may operate to bar the beneficiary's action or reduce the recovery. Because the mother is the beneficiary, her negligence would bar her recovery.

A foreseeable plaintiff must be within the zone of danger wherein a defendant would have reasonable foreseen harm on that person. A botched hair job is never expected to cost said persons spouse a promotion.

The correct answer is:No, because the man was not a foreseeable plaintiff within the "zone of danger." Discussion of correct answer:Under the majority view, a defendant owes a duty of care only to foreseeable plaintiffs who are within the "zone of danger"‚ that is, those to whom, under the circumstances, a reasonable defendant would have foreseen a risk of harm. Furthermore, the doctrine of proximate cause serves to limit a defendant's liability for unforeseeable damages. Liability is cut off where a court finds highly extraordinary the relationship between defendant's conduct and the resulting harm. Alternatively, many jurisdictions extend liability only to the foreseeable consequences of a defendant's negligence. Here, under either analysis, it is doubtful that the man's loss of a large promotion and corresponding loss of income would be considered a foreseeable consequence of the salon's botched hair-coloring session with the man's wife. As such, this is the best response. Incorrect. No, because the man was not a customer of the salon. Direct privity (as between a seller and purchaser, or between a provider of services and the recipient of these services) is not required in either strict liability actions or ordinary negligence claims. As such, the fact that the man himself was not a customer of the salon would not prevent his recovery in a negligence claim, provided the requisite elements were present.

If it's true, whether written or recorded, it is not defamation.

The correct answer is:No, because the photographer took a video of what he saw. Discussion of correct answer:One who publishes a statement of fact is not liable for defamation if the statement is true. Thus, in this case, as long as the video was not edited in any way, it would constitute a true representation of the pop star's actions. Therefore, the Web site would have an absolute privilege to publish the video without subjecting itself for liability for defamation. Incorrect. No, because the video was not taken with malicious intent. The pop star would be a "public figure" for defamation purposes, so a higher standard would apply to the photographer's intent. One who publishes a false and defamatory communication concerning a public figure or public official (relating to his official conduct) is subject to liability if he has knowledge that the statement was false or acts in reckless disregard as to its truth or falsity. However, truth is an absolute defense to defamation, as long as the video was not edited or otherwise altered to be misleading. Thus, this answer is incorrect.

A landowner may owe a child trespasser a higher standard of care under attractive nuisance doctrine.

The correct answer is:No, because the pizza shop owner owed a duty of reasonable care because it was foreseeable that the bus would attract a child trespasser. Discussion of correct answer:Generally, a landowner owes no duty of care to an unknown trespasser. Nor does a landowner have any duty to inspect to protect an unknown trespasser from injury. However, a landowner may owe a child trespasser a heightened standard of care. Under the attractive nuisance doctrine, a landowner has a heightened standard of care to protect children from dangerous artificial conditions conducted on the land. When the doctrine applies, the landowner has a duty to exercise reasonable care. Here, the facts indicate that the brightly colored bus in its current condition could pose an unreasonable risk of danger to children. The facts indicate the pizza shop is near a residential area, thus making it foreseeable that a child would be likely to trespass. Because of the child's age, he is unlikely to appreciate the risk and the risk of the danger of the bus outweighs its social utility. The pizza shop owner would owe the child a duty and would be subject to a heightened standard of care. Incorrect. Yes, because the child was warned of the danger. This answer is incorrect because the facts do not support the conclusion that the child would have been of suitable age to appreciate the danger even though there was a written sign. Because of the child's age, the sign warning of danger would likely be insufficient to satisfy the pizza shop owner's heightened duty to the child trespasser.

You can't blame the guy who sold you the home if he didn't build it - at least not on strict liability.

The correct answer is:No, because the real estate agency did not construct the house. Discussion of correct answer:Most jurisdictions hold sellers of mass-marketed new residences strictly liable for defects in those homes. There is a split in authority as to whether other new home sellers, such as custom builders who construct a few houses at a time or building contractors who construct residences under specific contracts, may be held strictly liable for defects in those constructions. However, no court has imposed strict liability on a defendant who sold a home he did not construct. Here, the real estate agency did not construct the home that it sold to the buyer. Therefore, the real estate agency will not be strictly liable to the man for the defects in his new house, and this answer choice is correct. Incorrect. Yes, because the real estate agency sold the house to the buyer as part of mass-marketed new residences. Although most jurisdictions hold sellers of mass-marketed new residences strictly liable for defects in those homes, no court has imposed strict liability on a defendant who sold a home he did not construct. Therefore, regardless of whether the house is part of a group of mass-marketed residences, the agency will not be strictly liable for defects in the house.

A landowner has no duty of care to an unknown trespasser, nor does a landowner have a duty to inspect his land for unknown trespassers. So if an unknown trespasser wanders onto your land and gets bit by your cat, not liable.

The correct answer is:No, because the runner was a trespasser. Discussion of correct answer:A defendant can be strictly liable for personal injuries inflicted by his animal if it has known dangerous propensities. The classic example of a domestic animal with known dangerous propensities is a dog that has previously bitten someone. In this case, the facts indicated that the homeowner's cat had a known dangerous propensity, in that the cat had previously bitten the homeowner's neighbor. However, most jurisdictions do not impose any liability for injuries to unknown trespassers inflicted by defendant's animals while they are on a defendant's land, even as to animals with known dangerous propensities. In this case, the runner was an unknown trespasser bitten while on the homeowner's land, and the homeowner was not aware of the runner's presence. As such, the runner will not prevail in the action. Incorrect. Yes, because the homeowner should have placed warnings about dangerous conditions on his land. A landowner has no duty of care to an unknown trespasser, nor does a landowner have a duty to inspect his land for unknown trespassers. As such, this answer is incorrect.

In an attractive nuisance question, we are not dealing with the issue of whether a child is negligent, but rather whether the child subjectively appreciates the danger that exists.

The correct answer is:No, if the boy recognized the risk that was involved. Discussion of correct answer:Here's an extremely tricky Multistate question dealing with the attractive nuisance doctrine. Under the attractive nuisance doctrine a possessor of land is subject to liability for physical harm to trespassing children where: (1) an artificial dangerous condition exists on the property; (2) the possessor knows or should know that children are likely to trespass on the property; (3) the children because of their youth, age, or immaturity fail to appreciate the danger or realize the risks involved in intermeddling with it; and (4) the utility of maintaining the dangerous condition is slight compared to the risks involved. It is important to note that the inability of a child to appreciate the danger is a subjective test. Therefore, choice (D) is preferred. In general, for this subjective test, the child, because of his lack of age and maturity, either must not appreciate the danger involved or must not discover the condition. Choice (C) is incorrect because it addresses the objective standard of conduct to which a child must generally conform in order not to be held liable for negligence. In an attractive nuisance question, we are not dealing with the issue of whether a child is negligent, but rather whether the child subjectively appreciates the danger that exists. Discussion of incorrect answers: Incorrect. Yes, if the tree house constituted an attractive nuisance.

Emergencies don't relieve individuals of acting reasonably. So no recovery unless boy acted unreasonably.

The correct answer is:No, unless the boy acted unreasonably when he pushed the neighbor off the go-cart. Discussion of correct answer:An individual, when engaged in any activity, has a legal duty to act as a reasonably prudent person under the circumstances. Note that a defendant has a duty to act as a reasonably prudent person would in the particular circumstances under which the defendant acted. The existence of an emergency does not relieve the individual from the responsibility of acting reasonably. Minors are expected to act in a manner that is similar to how a reasonable child of the same age, education, intelligence, and experience would have acted. Given the facts of the question, this is the best answer, and the neighbor's parents will not recover damages from the boy's parents unless the boy acted unreasonably when he pushed the neighbor off the go-cart. Incorrect. No, if the boy pushed the neighbor off the go-cart because he honestly believed that the neighbor was in imminent danger. An honest belief of imminent danger would not relieve the boy of liability if he acted unreasonably in pushing the neighbor off the go-cart. The test in this case is one of reasonableness.

For false imprisonment, must have intended to imprison.

The correct answer is:No, unless the clerks knew the businessman was still in the dressing room at the time they locked the store. Discussion of correct answer:To be liable for false imprisonment, the clerks working for the store must have desired to confine the businessman or known that there was a substantial certainty that he would be confined. If the clerks actually knew the businessman was still in the dressing room when locked the store, it was not only substantially certain, but inevitable, that the businessman would be confined. Incorrect. Yes, because reasonable clerks would have checked the dressing room before leaving the premises. The clerks should have known that a customer entering a dressing room shortly before closing might still be there. Reasonable clerks would know that if they did not check the dressing rooms, there was a risk someone would be left in the store. Some risk, however, is not sufficient for false imprisonment; some risk is sufficient for negligence. Because this question about a false imprisonment cause of action, this answer is not correct.

1 in 1000 person being affected is not a nuisance, i.e. not interfering with public at large.

The correct answer is:Only 1 person in 1,000 is affected by the vapor. Discussion of correct answer:The best defense to a cause of action in nuisance is to show that it is not a nuisance. If only 1 person in 1,000 is affected by the vapor, the plant is not interfering with a right of the public at large. Therefore, this is the best answer Incorrect. The plant was built long before the city became a retirement community. The fact that surrounding owners moved onto their properties after the company was there would be only one factor in determining whether an activity was a nuisance. The company could still be held liable. To argue that this is not a nuisance would be a better defense.

A serious reaction such as death always requires a warning label. Dizziness not so much.

The correct answer is:Only the woman will prevail, because the company did not include a warning about serious illness on its product. Discussion of correct answer:Whether absence of a warning of allergic or other adverse reactions is a dangerous defect depends upon the severity of the potential reaction and the number of people expected to be affected. A small number of reactions justify a warning if the adverse reaction threatens death or serious illness. A mild reaction might require a warning if a substantial number of people likely will experience it. In this case, the company should have included a warning about the serious illness, because that is a severe side effect, even though very few people were likely to experience the reaction. As such, the woman will prevail. Incorrect. Both the man and the woman will prevail, because the company did not include warnings about either dizziness or serious illness on its product. This answer is incorrect, because the man will not prevail. The side effect experienced by the man was too mild and was likely to be experienced by too few individuals. As such, the company did not need to place a warning about the side effect on its product.

Truth is an absolute defense to a defamation claim. A showing of untruth is essential to the success of a defamation case.

The correct answer is:Roger is a retired bus driver who has never been involved in any real estate transactions. Discussion of correct answer:Truth is an absolute defense to a defamation claim. If Roger has never been involved in any real estate transactions, then Alex's statement could not be true. A showing that Alex's statement is not true is essential to the success of Roger's action. Incorrect. Alex holds the public office to which Roger is seeking to be elected the following term. The fact that Roger is running for public office makes him a public figure, meaning that in order to recover in a defamation action against Alex, he must establish that Alex acted with malice. As such, Roger's case will be more difficult than if he were a private figure. Furthermore, the mere fact that Alex had a motive for defaming Roger is insufficient to establish the elements of defamation. As such, Roger's claim would not be significantly strengthened by a showing that Alex had a motive for defaming Roger, and, in fact, Roger's case would be made more difficult by the fact that as a public figure, Roger would be required to prove actual malice on Alex's part. Thus, this answer is incorrect.

Gorilla walks around, someone runs away from it and gets hurt. Owner liable.

The correct answer is:She will prevail, because the husband and wife are strictly liable for harm resulting from the escape of the gorilla. Discussion of correct answer:According to Restatement of Torts, 2d, Section 507, "A possessor of a wild animal is subject to (strict) liability to another for harm done by the animal to the other, his person, land or chattels, although the possessor has exercised the utmost care to confine the animal, otherwise to prevent it from doing harm." This fact situation is analogous to Restatement Illustration 1, Sec. 507, p. 13, in which a pet chimpanzee escaped from defendant's property and approached a group of children. The mother of one of the children, erroneously thinking the chimpanzee was about to attack her child, rushed to her child's assistance, and in her hurry and excitement, she stumbled and fell, breaking her leg. In accord with the position taken by the Restatement, the owner of the chimpanzee is subject to strict liability. Note that choice (B) is wrong because, despite the fact that the defendant(s) may have exercised utmost care to prevent the animal's escape, they are nonetheless strictly liable. Choice (C) is incorrect because it is not necessary that the possessor should have reason to know that the particular animal possesses a dangerous propensity. Although he may reasonably believe that it has been so tamed as to have lost all of these propensities, nonetheless he takes the risk that at any moment the animal may revert to and exhibit them. Choice (D) is wrong because a wild animal not kept under effective control is dangerous to the personal security of others, not only because it is likely to attack them but also because the fear of an attack may cause injury to result. Incorrect. She will not prevail, because the gorilla was not responsible for actually causing the mother's injury.

Again, good faith basis is not enough when suing an attorney for malpractice. Must show actual damages, that you actually had a case.

The correct answer is:That but for the attorney's negligence, she would have recovered damages from the hardware store. Discussion of correct answer:To prevail in a negligence action, the plaintiff must show that the defendant's act caused her actual damages. Thus, in this case, the woman can recover against the attorney only if she can show that she would have proceeded with and won the underlying action against the hardware store if the attorney had not been negligent. If she would have lost the underlying suit, the attorney's negligence caused her no loss. Incorrect. That she had a good-faith claim against the hardware store that was lost due to the attorney's misconduct. A good-faith claim in the underlying lawsuit is not a sufficient basis for a legal malpractice action. The woman must show that she would have won the case in order to show that she has suffered damage from the attorney's misconduct.

A client must show she would have won but for the attorney's negligence. A good-faith claim in a lawsuit is insufficient for a legal malpractice claim. If she was sure to lose then no harm.

The correct answer is:That but for the attorney's negligence, she would have recovered damages from the market. Discussion of correct answer:To prevail in a negligence action, the plaintiff must show that the defendant's act caused her actual damages. Thus, in this case, the woman can recover against the attorney only if she can show that she would have proceeded with and won the underlying action against the market if the attorney had not been negligent. If she would have lost the underlying suit, the attorney's negligence caused her no loss. Incorrect. That she had a good-faith claim against the market which was lost due to the attorney's misconduct. A good-faith claim in the underlying lawsuit is not a sufficient basis for a legal malpractice action. The woman must show that she would have won the case in order to show that she has suffered damage from the attorney's misconduct.

Truth is a defense against defamation claim.

The correct answer is:That his claims are true. Discussion of correct answer:Truth is a complete defense to any claim of defamation. Therefore, if the drug dealer can prove that his allegations are true, this would provide him with a complete defense in a defamation action. Incorrect. That his claims are true, and he knows it from personal knowledge. Truth is a complete defense to any claim of defamation. Therefore, if the drug dealer can prove that his allegations are true, this would provide him with a complete defense in a defamation action. There is no need that the defendant has personal knowledge of the information.

Negligence per se only applies in an action if the law was intended to prevent the harm that occurred.

The correct answer is:The boy's mother will not prevail on grounds of negligence per se. Discussion of correct answer:The unexcused violation of a legislative enactment or administrative regulation which is adopted by the court as defining the standard of conduct of a reasonable man, is negligence itself, or as commonly phrased, negligence per se. The violation of a criminal statute constitutes negligence per se where the plaintiff can show that: 1) he was a member of the class of persons the statute was designed to protect; and 2) he suffered the type of harm the statute was designed to prevent. In such instances, statutory violation will be considered sufficient to prove duty and breach of duty, but a plaintiff is still required to show causation and damages. Here, the doctrine of negligence per se does not apply. For one, the statute involved was a civil ordinance, not a criminal statute. And secondly, the type of harm suffered by the boy (an injury occurring when tripping over a sprinkler spigot) was not type of harm that the ordinance was designed to prevent. As such, the boy's mother will not prevail on grounds of negligence per se. Incorrect. The boy's mother will prevail, under the attractive nuisance doctrine. The "attractive nuisance" doctrine holds that a possessor of land is subject to liability for physical harm to children trespassing thereon caused by an artificial condition upon the land if: 1) the place where the condition exists is one upon which the possessor knows or has reason to know that children are likely to trespass; and 2) the condition is one of which the possessor knows or has reason to know and which he realizes, or should realize, will involve an unreasonable risk of death or serious bodily harm to such children; and 3) the children, because of their youth, do not discover the condition or realize the risk involved in inter-meddling with it or in coming within the area made dangerous by it; and 4) the utility to the possessor of maintaining the condition and the burden of eliminating the danger are slight as compared with the risk to children involved; and 5) the possessor fails to exercise reasonable care to eliminate the danger or otherwise to protect the children. Here, the homeowner's sprinkler was not a condition which "will involve an unreasonable risk of death or serious bodily harm to children," as described in the second element, nor do the facts indicate that the homeowner knew or had reason to know that her young neighbors were playing in the sprinkler. As such, the boy's mother cannot recover under the attractive nuisance doctrine.

A plaintiff's lack of comparative negligence by itself isn't enough. Would help to show that def did not prevent unreasonable risk.

The correct answer is:The city failed to exercise due care by letting at least three days pass without repairing the signal. Discussion of correct answer:A city must behave as a reasonable municipality to prevent unreasonable risks of injury to the users of its streets. Tort immunity is not given to a municipality if the municipality's agents are aware that inaction could lead to harm. Failure to repair a signal at an intersection known to be malfunctioning for three days may well pose serious risks of injury and be unreasonable. Hence this is the strongest argument in favor of the plaintiff. Incorrect. The driver was behaving reasonably when he entered the intersection since he stopped and then carefully proceeded. The fact that the driver behaved reasonably precludes any claim that he was comparatively negligent. This is irrelevant, however, unless the city was itself negligent in causing the driver's injuries in the first place. The plaintiff's lack of comparative negligence by itself is not sufficient to support a judgment in his favor.

When you wreck (small item) you bought it where destruction rises to the level of conversion

The correct answer is:The customer is liable, and will owe the pub for the full value of the portrait. Discussion of correct answer:The customer committed the intentional tort of conversion. Conversion is an intentional act by a defendant that causes the destruction of or a serious and substantial interference with plaintiff's chattel. In determining the seriousness of the interference and the justice of requiring the defendant to pay the full value, the following factors are relevant: (1) the extent and duration of the defendant's exercise of dominion or control; (2) the defendant's intent to assert a right inconsistent with the other's right of control; (3) the defendant's good faith; (4) the extent and duration of the resulting interference with the plaintiff's right of control; (5) the harm done to the chattel; and (6) the inconvenience and expense caused to the plaintiff. Here, there is no question that the harm is serious, because the customer caused irreparable damage to half of a unique work of art. This would certainly be destruction rising to the level of conversion. The customer may have had fairly "playful" motives, but he nonetheless destroyed the portrait. The customer will be liable for the full value of the chattel at the time of the conversion. Incorrect. The customer is liable, and will owe the pub for the diminished value of the portrait. This answer choice correctly concludes that the customer is liable, but supplies the wrong rule for damages. For a trespass to chattels, the defendant would be liable for damage or diminished value of chattel. Here, however, the seriousness of the interference and destruction will make the customer liable in conversion, and he will owe the pub for the full value of the portrait.

If you're going to accuse someone is a thief to your boss, make sure no one else is around.

The correct answer is:The customer will win, because the cashier knew that the other customer could hear her statement to her boss. Discussion of correct answer:The cashier would be conditionally privileged to make a defamatory statement to her employer for the purpose of protecting the employer's interests, as long as the statement is made only to persons who need the information to protect the interest. Here, she made the statement knowing that another customer was standing nearby and would also hear the statement. Disclosing this information to the customer would do nothing to apprehend the accused customer if he were, in fact, a thief. This was an excessive publication that would cause the cashier to lose her privilege. Incorrect. The store will win, because the cashier was privileged to make the statement to her boss. The cashier was privileged to make the statement to her boss, but not under circumstances where persons who do not need the knowledge to protect the store's interests would also hear the statement. Therefore, this is not the best answer.

A defendant's negligence must be a cause-in-fact and proximate cause of the plaintiff's injuries for a negligence action.

The correct answer is:The defective wires were incorporated into the pool light despite the exercise of reasonable care by the light company. Discussion of correct answer:In order to establish that the light company was negligent, the neighbor needs to show that the company owed him a duty, that the duty was breached, that he suffered damages as a result, and that the company's negligence was a cause-in-fact and proximate cause of his injuries. In this case, if the light company showed that it exercised reasonable care in manufacturing the pool light, then one of the elements of the negligence action would be missing. As such, the neighbor would not prevail, making this answer choice the company's strongest defense. Incorrect. The homeowner's negligence was the proximate cause of the neighbor's injuries. The homeowner's negligence is a proximate cause of the neighbor's injuries, but that alone would not prevent the light company's acts from also being a proximate cause. As a foreseeable independent intervening force, the homeowner's negligence does not supersede the light company's negligence. As such, the defense stated in this answer choice seems to be insufficient.

Even if invited in, officials usually treated as licensee. Duty to Licensee - to fix or warn of known dangers.

The correct answer is:The developer, because he was not aware of the dangerous condition of the fire escape. Discussion of correct answer:This is the best answer, because it correctly states the developer's required standard of care toward the inspector. Although the developer invited the inspector onto the premises for the developer's business purposes, courts traditionally classify the inspector as a licensee because he is a firefighter. (This also applies to police officers.) The developer's duty to a licensee is to fix or warn of known dangers. He is not required to make his land safe for licensees and is not required to inspect to discover dangers, even if a reasonable inspection would have revealed them. If the developer was aware that the fire escape would be dangerous to someone the inspector's size, its condition would be a known defect about which the developer must issue a warning. Nothing in the facts shows that the developer knew of the unsafe condition. Incorrect. The developer, because he did not purposely conceal a defect with paint. Nothing in the facts shows that the developer, intentionally or with disregard to the consequences, painted over a known defect. Only if he knew that he had painted over a defective condition would he be required to warn the inspector, a licensee, of this condition.

Chain of distribution -strict liability for creators of a product, even if only parts of it. this is different from the consumer expectation standard which is employed to determine if a product has a design flaw (goes to manufacturer's liability).

The correct answer is:The elderly woman will prevail, because the scooter producer is in the chain of distribution. Discussion of correct answer:A person who assembles component parts into a finished product is strictly liable for defects in the components used. The manufacturer of the component part is also strictly liable for defects in the component. In this case, the scooter producer is in the chain of distribution, so it will be liable, even though the scooter producer was not at fault for the defective wheel. Incorrect. The elderly woman will prevail, because of the consumer expectation test standard. The consumer expectation standard is employed to determine if a product has a design flaw. Under the consumer expectation test standard, a product is in a defective condition unreasonably dangerous when it is more dangerous than would be contemplated by the ordinary consumer who purchases it, with the ordinary knowledge common to the community as to its characteristics. However, this test is inapplicable here, because the defective wheel was not a design defect. A design defect exists when a product is manufactured as the manufacturer intended but still presents a danger of personal injury or property damage to plaintiff. In this case, the defective wheel was not as the manufacturer intended, so it does not suffer from a design defect. As such, the answer is incorrect.

defamation does not require showing malice, least useful reason for why a defamation case would be dismissed or won.

The correct answer is:The ex-girlfriend could not prove that the columnist acted with malice. Discussion of correct answer:When suing for defamation, a plaintiff ordinarily does not need to prove that the defendant acted with malice. Malice must only be proven to defeat a qualified privilege or where the plaintiff is a public official or public figure. Here, the ex-girlfriend is a private figure. As such, she does not need to prove that the columnist acted with malice, and so her inability to do so would be the least helpful reason why the columnist's motion to dismiss was granted. Incorrect. The ex-girlfriend cannot show she has been negatively affected because of the publication. Defamation per se is a classification of defamation where it is apparent on the face of the statement that it would be harmful to the subject's reputation. The defamation per se categories are statements that the subject has committed a crime of moral turpitude, has a loathsome disease, has characteristics incompatible with her profession, or is unchaste (if female). The columnist's allegation that "Terribly Teary-eyed" has had experience as a prostitute is a statement of unchastity and qualifies as defamatory per se. Under the circumstances, the ex-girlfriend need not show special damages. Thus, this answer is incorrect.

If the drunkard got drunk and got into an accident, he was contributory negligent and this may save the bar if they try to pin his death on him for what happened in the hospital.

The correct answer is:The firefighter was contributorily negligent. Discussion of correct answer:Contributory negligence is a defense that may be raised against the plaintiff even when the defendant's duty of care has been established by statute. Some statutes specifically exclude the intoxicated person as a plaintiff under the dram shop act. Those that do not, presumably, allow the defendant to assert the plaintiff's contributory negligence as a bar to his recovery anyway. This answer is correct. Incorrect. The nurses' negligence constituted a superseding cause that relieved the bartender of liability. The nurses' acts will be held to constitute a superseding cause only if they were so unusual as to be unforeseeable and to break the chain of causation. This is a possible defense, but it is not the bartender's strongest defense because the question does not state any facts to indicate whether the nurses' failure to follow the doctor's orders was unusual. Medical malpractice is often deemed foreseeable. Thus, this answer is incorrect.

To prevail on malicious prosecution, the case must have been terminated in a manner indicating the guy was innocent. Getting out on a defect doesn't make you pure.

The correct answer is:The gang member should not prevail, because his case was dismissed on a technicality. Discussion of correct answer:To prevail on malicious prosecution, the case against the plaintiff must have terminated in a manner indicating the plaintiff's innocence. Malicious prosecution is the institution of criminal proceedings by a defendant, done for an improper purpose, and without probable cause, which terminate favorably to the plaintiff, and which cause the plaintiff damages. The criminal prosecution must have terminated in a fashion indicating that the plaintiff was innocent of the charges. Termination on the merits (i.e., an acquittal after trial or court dismissal for lack of sufficient evidence) is sufficient in this regard; termination based on procedural or technical defects, prosecutorial discretion, or similar grounds is not. A mistrial resulting from juror misconduct is a technical defect that does not indicate the gang member's innocence. Therefore, the gang member should not prevail in his action. Incorrect. The gang member should prevail, because the chauffeur lied to the police. The chauffeur's lie to the police satisfies one element of malicious prosecution. However, for the plaintiff to prevail, the prosecution against the plaintiff must also have terminated in a manner indicating the plaintiff's innocence. In this case, the gang member's case was dismissed on an unrelated technicality. Malicious prosecution is the institution of criminal proceedings by a defendant, done for an improper purpose, and without probable cause, which terminate favorably to the plaintiff, and which cause the plaintiff damages. Termination on the merits (i.e., an acquittal after trial or court dismissal for lack of sufficient evidence) is sufficient in this regard; termination based on procedural or technical defects, prosecutorial discretion, or similar grounds are not. A mistrial resulting from juror misconduct is a technical defect, which does not indicate the gang member's innocence. Therefore, although the chauffeur lied, the gang member should not prevail.

If it was not a breach of care to let something happen, you can't blame someone for it in a negligence case. This is a defense against a negligence case.

The correct answer is:The gardener should not be held liable, because the sprouting of poison ivy in her garden is not probative of a breach of due care. Discussion of correct answer:In an action for negligence, the plaintiff has the burden of proving that the defendant failed to conform to a standard of conduct appropriate for a person of the defendant's education and experience. It may be inferred that harm suffered by plaintiff is caused by negligence of the defendant only when (1) the event is of a kind that ordinarily does not occur in the absence of negligence; (2) other responsible causes, including the conduct of the plaintiff and third persons, are sufficiently eliminated by the evidence; and (3) the indicated negligence is within the scope of the defendant's duty to the plaintiff. Here, these elements are not satisfied. For one, it is questionable whether the existence of poison ivy in the gardener's garden is indicative of a breach of reasonable care on her part, inasmuch as poison ivy and other unwelcome plants may crop up anywhere in any lawn or garden despite a gardener's impeccable care. Thus, although the son's and, therefore, the neighbor's, injury was caused by poison ivy, it is not clear that the growth of the poison ivy was the result of a breach of due care on the gardener's part. As such, the gardener's best defense is that the harm suffered by the son was not the result of a breach of due care on the gardener's part. Incorrect. The gardener should not be held liable, because she had no reason to know that the son was allergic to poison ivy or a hemophiliac. The gardener's lack of awareness alone is insufficient to guarantee that she will not be held liable for the resulting damages. Regardless of her awareness of the son's conditions, if the gardener is found to have breached a duty of reasonable care in failing to prevent poison ivy from growing on her trellis, she will be liable for the resulting injury. As such, this is not the best argument in the gardener's defense.

Under the "shopkeeper's privilege," a store owner would therefore be privileged to act with reasonable force even if mistaken.

The correct answer is:The grocery shopper, but only if the store owner used force that was unreasonable under the circumstances. Discussion of correct answer:The store owner touched the grocery shopper when he grabbed him by the arm and shoved him into the office. However, under the "shopkeeper's privilege," the store owner had the right to use reasonable force to detain the grocery shopper for a reasonable time to investigate a possible theft. Under the circumstances, the store owner would have reason to believe that the grocery shopper may have been committing a theft. Therefore, the store owner would be privileged to act, provided he used reasonable force. So, the grocery shopper could recover only if the store owner used unreasonable force in detaining the grocery shopper. Incorrect. The grocery shopper, because he had permission to use the restroom. While the grocery shopper did indeed have the right to be making use of the bathroom and be in the storage room under the circumstance, the store owner reasonably believed that the grocery shopper was committing a theft. Under the "shopkeeper's privilege," the store owner would therefore be privileged to act with reasonable force. Whether the grocery shopper had permission to use the restroom is not decisive.

Can't eject someone if there is a raging blizzard going on outside or other life-threatening situation.

The correct answer is:The hiker, because the blizzard was a life-threatening situation. Discussion of correct answer:A defendant is permitted to injure a plaintiff's property if it is reasonably necessary to avoid a substantially greater harm to the public, to himself, or to his property. The hiker was privileged to trespass on the cabin owner's property because of the private necessity of saving himself from death or serious injury in the snowstorm. Because of this privilege, the cabin owner did not have the right to eject the hiker into the storm and committed battery when he did. Incorrect. The cabin owner, because the hiker refused to leave when asked. Because of the life-threatening storm, the hiker was privileged to be in the cabin even after the cabin owner asked him to leave. A defendant is permitted to injure a plaintiff's property if it is reasonably necessary to avoid a substantially greater harm to the public, to himself, or to his property.

Parents are not vicariously liable for the negligence torts of their children unless they did not use reasonable care in supervision.

The correct answer is:The husband and the wife will win, unless they did not act reasonably in permitting their daughter access to the car keys. Discussion of correct answer:Whenever a question addresses a situation where vicarious liability may apply, the student should also consider the potential liability of the defendant for his/her own negligence. While the husband and the wife may not be vicariously liable for the daughter's torts, because parents are not vicariously liable for the negligence torts of their children, they may still be liable in negligence if they did not use reasonable care in supervising the daughter's access to the car keys. Discussion of incorrect answers: Incorrect. The neighbor will win, because the husband and the wife are liable for the negligence of their daughter. Parents are generally not vicariously liable for the negligence of their children. They may only be liable for their own negligence in allowing the child to engage in the potentially dangerous activity.

In a comparative negligence jurisdiction, each tortfeasor can be held liable for the portion of the plaintiff's damages that the tortfeasor caused. This does not mean seeking indemnity from both.

The correct answer is:The lawyer can obtain contribution from the sensor manufacturer if the sensor was defective. Discussion of correct answer:In a comparative negligence jurisdiction, each tortfeasor can be held liable for the portion of the plaintiff's damages that the tortfeasor caused. The effect of joint and several liability (i.e., where the tortfeasors may be sued separately or all together) is that each defendant acting in concert with the other is liable for the entire amount of the plaintiff's damages. While the plaintiff is not required to join two or more defendants, the named defendants may implead the remaining tortfeasors for contribution. Where joint tortfeasors have together caused a single plaintiff a single harm, there is a right of contribution among them, even though judgment has not been recovered against all or any of them. Here, if the sensor manufacturer produced a defective product, the lawyer may obtain appropriate contribution from the manufacturer. Incorrect. The lawyer is entitled to indemnity from both the dog trainer and the sensor manufacturer. This is not the correct answer. One of two or more defendants responsible for the plaintiff's injuries may, in some situations, cause one or more of the other defendants to satisfy the entire amount of the plaintiff's damages. Where one defendant is vicariously liable for the tort of another, directly liable defendant, the first defendant may recover the entire amount of any damages paid to the plaintiff from the second defendant, who was actually responsible for the plaintiff's injury. Also, some contractual relationships may involve indemnification clauses. Here, because there is no indication of any contractual indemnification, nor any vicarious liability, indemnity is not available.

If you ask a stranger for advice, and the stranger misrepresents to your detriment then stranger can be held liable.

The correct answer is:The man will prevail, because his reliance on the shopper was justified. Discussion of correct answer:An intentional misrepresentation by a defendant, made with scienter, which is material and justifiably relied upon by the plaintiff and which causes damages to the plaintiff, is actionable. If the defendant has an interest that he fails to disclose to the plaintiff, it is justifiable for the plaintiff to rely on the defendant's expression of opinion on the subject of that interest. In this case, the shopper made a false, affirmative statement that he knew to be false and yet failed to disclose to the man that he owned part of the painting company. Thus, the man's reliance on the shopper was justified and the man will prevail in his suit. Incorrect. The man will not prevail, because his reliance was not justified since the shopper was a stranger. Generally, relying on an opinion of a person who owes no fiduciary duty to a plaintiff is not justifiable. However, when the defendant does not disclose that he has an interest, the reliance will be deemed justifiable. In this case, the shopper did not disclose his interest, so the man's reliance was justified.

Where nuisance is concerned, if it only affect 1 in 1000 persons then not sufficient nuisance claim. The plaintiff cannot, by devoting her own land to an unusually sensitive use, recover for nuisance.

The correct answer is:The manufacturer, because the breeder's business is abnormally sensitive to harm caused by the high-frequency sound waves. Discussion of correct answer:This is a tricky Torts question because the breeder is asserting an "action," which could be either trespass or nuisance. Thus, it is necessary to carefully read each answer choice in addressing both possible causes of action. Choice (A) is incorrect because "coming to the nuisance" is never an absolute defense in a nuisance action. Rather, it is merely one factor the court will take into consideration. Choice (C) is incorrect because trespass requires the entry or invasion of a tangible thing or person. Although invisible gases and microscopic particles will suffice, lights, noises, and vibrations are usually not considered sufficient. Choice (D) is incorrect because in nuisance actions, the disturbance must be annoying, offensive, or inconvenient to a normal person in the particular locality. The plaintiff cannot, by devoting her own land to an unusually sensitive use, recover for nuisance. Therefore, choice (B) is the best answer, because normal persons in the community are not offended by the sound waves as they are not audible to people. Incorrect. The manufacturer, because the breeder came to the nuisance after the factory had already been in operation for a number of years.

Animal Noises on a farm are normal, not a nuisance. I got this question wrong because I am a city slicker.

The correct answer is:The neighbor will not prevail, because the presence of animals in farming areas is not unreasonable. Discussion of correct answer:A private nuisance is substantial and unreasonable interference with a plaintiff's use and enjoyment of her land. To recover in a nuisance action, the interference with a plaintiff's use and enjoyment must be "substantial," that is, it must be offensive, inconvenient, or annoying to an average person in the community. A plaintiff cannot, by devoting her land to an unusually sensitive use, make a nuisance out of conduct that would otherwise be relatively harmless. Here, the neighbor lived adjacent to the nature lover's property for a long time and had no complaint about her animals until he was unable to sell his land. Moreover, inasmuch as they lived in a farming community, it is unlikely that the nature lover's conduct would be offensive, inconvenient, or annoying to an average person in the community. Whether the nature lover kept the animals well away from the property lines will not matter to the court as much as whether the animals produced some type of significant interference with the neighbor's use or enjoyment of his land. Because it is not unreasonable to raise animals on a farm, it is unlikely that the neighbor can prevail in a nuisance action. Incorrect. The neighbor will prevail, if he can show that the nature lover does not keep the animals away from the edge of the property. Whether the nature lover's animals interfered with property lines will not matter to the court as much as whether the animals produced some type of significant interference with the neighbors use or enjoyment of his land. Because it is not unreasonable to raise animals on a farm, it is unlikely that the neighbor can prevail in a nuisance action.

Again, if the message is true it's not defamation. If the officer arrested you in drag and you say he likes going around the mall in drag, close enough.

The correct answer is:The officer should prevail, unless he was disguised as a woman when he arrested the suspect, or the suspect's statement is true. Discussion of correct answer:The officer is likely to prevail unless the suspect has the benefit of an absolute privilege in connection with the judicial proceedings, or the suspect has the truth as a defense. As a matter of law, where an absolute privilege applies, a defendant may not be held liable for an otherwise defamatory message. A participant in judicial proceedings (i.e., the judge, attorney, witness, or juror) is not liable for any defamatory message that is reasonably related to the proceedings. Therefore, if the officer was disguised as a woman when he arrested the suspect, the suspect's statement would have been reasonably related to the proceedings. Historically, a majority of jurisdictions regarded proof that a defamatory message was true as a complete defense to a defamation action, even if the defendant acted out of ill will and even if he was not aware that the defamatory message was true. A minority of jurisdictions impose liability on a defendant who publishes a true defamatory message if he generally acted out of improper motives, ill will, or spite. Therefore, under the majority rule, the suspect could not be liable for a true statement. Incorrect. The officer should prevail, even if the suspect's statement is true, if the suspect did not know it to be true. Truth is a complete defense to a claim of defamation, regardless of whether the defendant knows that the statement is true. Historically, a majority of jurisdictions regarded proof that a defamatory message was true as a complete defense to a defamation action, even though the defendant acted out of ill will and even if he was not aware that the defamatory message was true. A minority of jurisdictions impose liability on a defendant who published a true defamatory message if he generally acted out of improper motives, ill will, or spite. Therefore, under the majority rule, the suspect could not be liable for a true statement, regardless of whether he knew the statement was true.

If a character is acting shifty, this is a sign of a negligence actions if there was no other reasonable action taken.

The correct answer is:The owner of the facility will be liable for negligence, if a reasonable storage facility operator would have contacted the police based upon the thief's odd behavior. Discussion of correct answer:A commercial bailee is not liable for conversion if he returns the chattel to his bailor without knowledge of any other claims to the property. This rule is necessary to make commercial bailment business possible. If the bailee was potentially liable for conversion every time he returned a chattel to the party who had placed it with him, the investigation burdens on bailees would make the business economically impossible. Because the facility owner did not know that the car was stolen, he could only be liable if he was negligent in how he dealt with the car. Incorrect. The owner of the facility will not be liable. Although this answer acknowledges the facility owner's protection from liability for intentional tort, the facility owner might still be liable in negligence if he did not behave as a reasonable person under the circumstances.

Public Nuisance requires a harm different from that of the community at large.

The correct answer is:The plaintiffs will prevail only if they can show that they suffered some damage different from the public at large. Discussion of correct answer:A private person can file a claim for public nuisance if he sustained a harm different from that of the community at large. If the homeowners can show that they suffered a harm different from that of the community at large, they can prevail against the company under a public nuisance theory. Incorrect. The plaintiffs will prevail, if the company's activity interfered with anyone's private use of the plaintiffs' property. Public nuisance addresses unreasonable interference with rights common to the general public, not the interference with private use of the land, which is a private nuisance concern. Therefore, this is not the best answer.

Only when conditions are abnormally dangerous would strict liability rise for harm on a trespasser. Otherwise, where a trespasser is foreseeable, reasonable care is owed to prevent foreseeable harm.

The correct answer is:The restoration company is liable, because the danger to children from objects falling from the scaffolding was foreseeable. Discussion of correct answer:Although the girl is a trespasser, the restoration company still owes her a duty of reasonable care if it was foreseeable that children would trespass, and that there was an unreasonable risk of harm. The scaffolding was in an area attractive and accessible to children, and posed great danger to those who climbed onto the scaffolding as well as those on the ground below. Because the danger to young children was great, and the cost of erecting a fence or providing a guard would almost certainly not be great, a judgment for the parents would likely be due to the foreseeability of objects--including people--falling from the unprotected scaffolding. Therefore, this answer is correct. Incorrect. The restoration company is strictly liable for the dangerous condition it created. Strict liability may be imposed on the creator of an abnormally dangerous condition that is not usually present in that area. It is common to erect scaffolding around structures that are being repaired, so this would not be considered an abnormally dangerous activity.

Absolute privilege applies to government officials on the legislative floor where it concerns speech.

The correct answer is:The senator will prevail, because he was privileged to make the statements. Discussion of correct answer:Where an absolute privilege applies, a defendant may not be held liable for an otherwise defamatory message as a matter of law. A legislator (federal, state, or local) is not liable for a defamatory message uttered while on the floor of the legislature or during hearings or committee proceedings. The nature or content of the defamatory message or its relationship to any matter before the legislature is immaterial to availability of this privilege. Here, because a senator made the statements on the floor of the Senate, the absolute privilege applies, regardless of whether the statements were defamatory. Incorrect. The senator will prevail, if he reasonably relied on the newspaper article. Any person who repeats a defamatory message is liable as a publisher, even where the repetition is qualified by such terms as, "alleged" or is said not to be the opinion of the publisher. However, where an absolute privilege applies, a defendant may not be held liable for an otherwise defamatory message as a matter of law. Legislators (whether federal, state, or local) are not liable for defamatory messages uttered on the floor of the legislature or during hearings or committee proceedings. The nature or content of the defamatory message or its relationship to any matter before the legislature is immaterial to availability of this privilege. Therefore, the senator will prevail, but it will be on the basis of legislative privilege, not reasonable reliance on the newspaper. Thus, this answer is incorrect.

If you caused somoene to fall into a ditch, you will be liable to him if you don't go for aid.

The correct answer is:The worker fell into the trench when he unexpectedly stepped in front of the man, who was riding a bicycle. Discussion of correct answer:Generally speaking, a bystander has no duty to come to the aid of someone in peril. However, if the bystander has created the peril, even if utterly accidentally and without intent or negligence, the bystander has a duty to use reasonable care to attempt a rescue. This is the modern approach, which is still a minority amongst jurisdictions. Thus, here, if the man, even through no fault of his own, caused the worker to fall into the ditch, the man would have a legal obligation to attempt to go to the worker's aid. Incorrect. The worker is the man's aunt, who tripped into the trench while being chased by a mugger. A bystander has no duty to come to the aid of someone in peril unless he put that person in peril or another exception applies. Although parents owe a special duty of care toward their children, this does not extend to nephews and aunts.

Teachers have a duty to protect bratty students. Affirmative Duty stemming from special relationship.

The correct answer is:The teacher should be held liable for negligence. Discussion of correct answer:The teacher's teacher/student relationship with the boy gives rise to an affirmative duty to protect the boy from injury. This special relationship between defendant and plaintiff takes place when the defendant derives economic benefit or when the defendant occupies a position of power over the plaintiff. The following relationships have been generally recognized as triggering the duty to care for the plaintiff: (1) employer/employee (during and in the scope of employment); (2) common carrier and innkeeper/customer; (3) school/pupil; (4) parent/child; and (5) jailer/prisoner. Consequently, the teacher breached her duty to the boy when she failed to warn the boy of the dangerous stairs. Incorrect. The teacher will not be held liable for negligence, because the teacher did not cause the boy's injury; the fall down the stairs did. A defendant's conduct is considered the cause-in-fact of an event if that event would not have occurred but for the existence of the conduct. Here, it can be said that but for teacher's failure to warn the boy of the condition of the stairs (an affirmative duty that the teacher had due to his special teacher/student relationship with the boy), the boy would not have fallen down the stairs. As such, it is not true that the teacher's omission was not a cause-in-fact of the boy's injuries. Thus, this answer is incorrect.

Eggshell Plaintiff: you take them as you get them. If you hurt someone with a present condition you are still fully liable to them.

The correct answer is:The testimony will not have any impact on the driver's liability. Discussion of correct answer:When a defendant's conduct risks physical injury to a foreseeable plaintiff, the defendant will be liable for the full extent of the physical injury even if that extent was unforeseeable. This is known as the "thin-skulled plaintiff" rule, or that the defendant "takes his plaintiff as he finds him." When a defendant acts in a manner that risks physical injuries to others, he bears the risk that the plaintiff has some pre-existing condition that will result in more extensive injuries than the typical plaintiff would have manifested. Incorrect. The worker's recovery will exclude those injuries resulting from the fall. The fall that the plaintiff experienced before the tort is irrelevant to the calculation of damages. This answer choice seems to imply that the fall will affect liability.

Manufacturing Flaw is Strict Liability. Negligent conduct does not bar recovery under strict liability.

The correct answer is:The woman should recover, because the product was defective. Discussion of correct answer:A product manufactured in a form other than the manufacturer intended contains a manufacturing flaw. Here, the hair spray was defective in that it contained too much alcohol. All jurisdictions impose strict liability on the manufacturer and all other sellers of the product in the chain of distribution for personal injury or property damage to plaintiff caused by a manufacturing flaw. The warning here did not prevent the product from being unreasonably dangerous. Furthermore, a foreseeable misuse does not bar recovery under products liability, even if a warning label was placed on the product. (It may reduce recovery in some jurisdictions.) Thus, the fact that the woman did not heed the written warning does not bar her recovery, because the manufacturer could expect that some people would smoke while using the product. As such, the woman should recover in strict products liability. Incorrect. The woman should not recover, because the can had a warning against spraying near an open flame. Negligent conduct on the part of the plaintiff does not bar recovery under strict products liability if the misuse by the plaintiff was nevertheless reasonably foreseeable, although it may reduce damages in some jurisdictions. Here, the hair spray manufacturer could expect that some people would smoke while using the hair spray, despite the warning label. Furthermore, the facts state that the hair spray had a manufacturing defect in that it contained too much alcohol. Any seller of a product that is unreasonably dangerous due to a manufacturing defect is subject to strict liability.

A foreseeable danger exacerbated by third party does not alleviate negligence of manufacturer.

The correct answer is:The woman's negligence was a foreseeable danger the risk of which was increased by the manufacturer's negligence. Discussion of correct answer:The woman's negligence in driving the car knowing of the potential problem is an independent, intervening force which contributed to the man's injuries, but it does not relieve the manufacturer of liability because it is the type of danger the risk of which would foreseeably be increased by the manufacturer's negligence. Discussion of incorrect answers: Incorrect. The woman's negligence was the proximate cause of the man's injuries. This choice correctly labels the woman's negligence, but it fails to identify its significance in the litigation. An event may have more than one proximate cause, and the woman's negligence in driving the car while knowing of the brake problem was an independent, intervening force which was a proximate cause of the man's injuries (i.e., The man could recover from the woman). However, because the danger of negligent driving is a foreseeable danger, it is a danger which the manufacturer must guard against by exercising due care in the manufacture of its cars. But this choice does not focus on that aspect of the woman's negligence and for this reason it is not the best choice.

Where structures and fixtures are concerned, it is strict liability or nothing. Reasonable care does not apply. The plaintiff collects because a building defect caused it or she doesn't.

The correct answer is:The woman, because a defect in the residence caused her injury. Discussion of correct answer:In most jurisdictions, the owner-developer of tract housing is subject to strict liability if an injury results from an unreasonably dangerous defect in a house he constructed. Incorrect. The woman, because the builder failed to use reasonable care in installing the fireplace insert. The builder will be liable on a strict liability theory. Therefore, whether or not he used reasonable care is not the proper analysis in this question. Thus, this answer is incorrect.

If you own a place and smell a gas leak, put up a warning or you may be liable if that would have reduced the risk.

The correct answer is:Yes, if a warning to persons entering the restroom would have reduced the risk. Discussion of correct answer:The businessman owes a duty of reasonable care to his employees. Because he recognized the risk of the gasoline fumes but had few options for getting assistance at that time, a reasonable person might well have posted a warning to minimize the risk of injury. Therefore, this is the best answer. Incorrect. No, because the injury was due to circumstances beyond the businessman's control. The creation of the hazard was beyond the businessman's control, but taking steps to end the hazard or minimize the risk it posed was within the businessman's control. Therefore, this is not the best answer.

A one-time seller (ditching couch on amazon) is not held up to strict liability where it concerns product defects (maybe other torts, just not strict liability).

The correct answer is:The young man prevails, because he was a one-time seller. Discussion of correct answer:In general, if strict liability is otherwise applicable, any plaintiff injured while using a defective product may recover damages from an appropriate defendant. However, occasional or one-time sellers are not proper defendants for purposes of strict products liability because they are not in the position to further the goals of the tort—safer products and cost spreading. The young man is not a commercial supplier, a retailer, or a distributor. Instead, he just sold one couch because he was moving to a new apartment. As such, he is a one-time seller and is not a proper defendant in a strict products liability suit. He may still be liable for other torts, but not strictly liable. Discussion of incorrect answers: Incorrect. The grandmother prevails, because the young man did not warn the woman about the broken leg. Although the young man did not warn the woman about the broken leg, the young man may still not be sued in a strict product liability action, because he is only a one-time seller. The young man may still be liable, but not strictly liable.

A rescuer of someone endangered by a flawed product who then gets hurt can bring claim against the negligent cause.

The correct answer is:Yes, because he was a rescuer. Discussion of correct answer:There is a modern trend to permit one not using a defective product to invoke strict liability against an appropriate defendant if it was reasonably foreseeable that such a plaintiff might be injured by the defective product. Examples include bystanders and rescuers. In this case, the driver was a rescuer and was injured while attempting to rescue the man who was endangered by a defective tire manufactured by the defendant. As such, this is the correct answer. Incorrect. No, because the driver assumed the risk when he decided to pull the man out of the wreckage. As initially conceived, the plaintiff's unreasonable conduct was not a defense to strict products liability unless the plaintiff knew of the defect, comprehended the risks posed by the defect, and voluntarily elected to expose himself to those risks. That is, only assumption of the risk was a defense to strict products liability. In this case, the driver did not know of the defective tire, did not voluntarily expose himself to using the tire and in either case was a rescuer. Hence, he cannot be deemed to have assumed the risk.

If the door was locked, privacy was expected.

The correct answer is:Yes, because her hairdresser locked the doors before starting to work. Discussion of correct answer:One who intentionally intrudes, physically or otherwise, upon the solitude or seclusion of another or his private affairs or concerns, is subject to liability to the other for invasion of his privacy if the intrusion would be highly offensive to a reasonable person. Unlike all other forms of the tort of invasion of right of privacy, the tort of intrusion into seclusion does not require publication. The hairdresser's locking of the doors before beginning to color the celebrity's hair indicates the celebrity's desire to seclude herself. In fact, even if the doors had not been locked, taking photographs of the celebrity while she was inside a private home would most likely still be considered an unreasonable intrusion upon her solitude. Therefore, not only would the celebrity most likely get an injunction against publication, she might also be successful in suing for damages even if the photographs were not published. As such, this is the best answer. Incorrect. No, because she was not in her own house. Seclusion can be anywhere the plaintiff has a right to physical solitude or to the privacy of personal affairs or concerns. Given that the celebrity was inside the private home of someone she knew and not out in public, she is entitled to bring an action for intrusion into seclusion.

If someone can overhear the statement when said out loud, it can qualify as defamation.

The correct answer is:Yes, because it was reasonably foreseeable that someone would overhear the statement. Discussion of correct answer:To prevail in a defamation action, a plaintiff must establish that a defendant published defamatory material concerning that plaintiff, which caused damage. The message must be communicated to a third person. While the defamatory message must be understood by the person who receives it, the recipient need not believe the defamatory message or have a rational reaction. If it is reasonably foreseeable that a third party might overhear a defamatory message, and one does so, there is sufficient publication to bring suit. Given that the male investment banker yelled out his defamatory remarks at a crowded lunch spot, it was reasonably foreseeable that his statement might be overheard. Therefore, there was publication, and the female investment banker is likely to prevail. Incorrect. No, because there was no publication. There is sufficient "publication" of a defamatory statement if it was reasonably foreseeable that a third party would hear it, and one does. Here, it was reasonably foreseeable that the male investment banker's statement might be overheard by others, because he yelled it at a crowded restaurant, and some did. Therefore, there was publication, and the female investment banker is likely to prevail.

Even with Alzheimer, that woman can still held accountable for intended offensive contact.

The correct answer is:Yes, because the elderly woman intended to cause a harmful or offensive contact. Discussion of correct answer:Battery requires an intentional act that causes a harmful or offensive contact with the person of the plaintiff or something closely associated with the plaintiff's person. The requirement of intent is satisfied if the defendant either intended to cause a harmful or offensive contact or knew that a harmful or offensive contact was substantially certain to occur. A mentally incompetent person can form the requisite intent to commit a battery or any other intentional tort. While a mentally incompetent person can have the requisite intent, the person's impaired mental abilities can bear on the question whether the necessary intent was formed. The facts reveal the elderly woman had dementia but there are no facts which suggest she was unable to form the necessary intent to commit a battery. Here, the facts suggest that she intended to cause a harmful contact because she was upset with the efforts of the nurse to change her adult diaper. Incorrect. No, because the elderly woman could not form the intent to commit battery. This answer is incorrect because the facts do not support the conclusion that the woman was unable to form the intent necessary to commit battery. It is true that a person's lack of mental capacity could affect the existence of intent, but generally an incompetent person is not prevented from forming the requisite intent to commit a battery or any other intentional tort simply because of incompetency. The facts indicate that while the elderly woman had dementia, she had the desire and intent to cause a harmful contact because she did not want the nurse to change her diaper.

If it was not actually the employee, but his on-site daughter, then duty scope isn't the issue. Just negligence, which can be imputed on employer.

The correct answer is:Yes, because the employee's negligence will be imputed to his employer. Discussion of correct answer:An employer is liable for his or her employees' tortious acts that are committed within the scope of employment and that cause injuries or property damage to a third person. If an employee negligently entrusts performance of his work to a third person, the employer may be liable by imputed negligence. In this case, it was the daughter who was driving the car, and there is nothing to indicate that she was an employee of the computer company. However, given that the employee breached his duty of due care by having his young daughter drive him to his next service call, the employee's negligence may be imputed to his employer, and the computer company will be liable for the resulting damages. As such, this is the best answer. Incorrect. Yes, because the scope of the employee's employment includes in-home tech support. It is true that the scope of the employee's employment included in-home tech support. However, because it was not the employee but the daughter, a non-employee, who was driving the vehicle that hit the client's car, the analysis of the computer company's liability cannot end there. The better answer is the one that describes the legal mechanism by which the computer company will be liable.

Again, as with escaped wild animals, escaped children on motor vehicle are responsible for indirect harm.

The correct answer is:Yes, because the girl did not behave as a reasonably prudent child of the same age, knowledge, intelligence, and experience under the circumstances. Discussion of correct answer:As a 10-year-old child, the girl's conduct would be judged against the standard of what a reasonable child of the same age, knowledge, intelligence, and experience would have done under the circumstances. However, if the girl was engaged in an adult activity, she would be held to the tougher objective standard of what a reasonable adult person would have done under the circumstances, with no allowance for her limited maturity. Because operating a toy boat would likely not be considered an adult activity, this would be the applicable standard. Incorrect. No, because her harm was caused by her boyfriend. Even though the boyfriend was the direct cause of the girlfriend's injury, the 10-year-old girl was also a joint tortfeasor, as her actions contributed to causing the girlfriend's injury. The girlfriend would be able to sue any joint tortfeasor until she collects a full satisfaction of her injuries.

Joint Venture similar to partnership. In a partnership each member is vicariously liable for torts of other member.

The correct answer is:Yes, because the investor and the club manager entered into a joint venture. Discussion of correct answer:When two or more individuals agree to enter into an undertaking in the performance of which they have a community of interest and mutual right of control, they are said to be engaging in a joint venture. Because such an arrangement is similar to a business partnership, each member is vicariously liable for any torts committed by the others within the scope of the enterprise. Here, the investor and the club manager jointly created the nightclub. The club manager provided the use of his loft, and the investor provided the start-up capital. That they both contributed to the business venture from which they would both benefit shows their joint undertaking. The investor and the club manager had a community of interest in the nightclub. As host, the investor was responsible for maintaining a well-ordered environment. While the investor initially attempted to limit the scope of his employment, he and the club manager subsequently agreed to add to the investor's duties by his managing the club on first Friday evenings. Because they agreed that the investor would run the club on certain occasions, his duties, risks, and liabilities also expanded. Both parties had a mutual right of control in the nightclub operations when they formed a joint venture. Because the investor was acting within the scope of his employment agreement and within the scope of the enterprise when he escorted the man from the nightclub and tossed him on the street, the club manager will be vicariously liable for any torts committed by the investor. Therefore, the club manager will be vicariously liable for the man's injuries and damages. This is the correct answer. Incorrect. Yes, because the injury occurred on the club owner's property. A possessor of land who holds it open to the public has a duty to members of the public who enter in response to his invitation. Thus, the club manager, as possessor of the property, would owe a duty to (1) inspect the loft premises and (2) make the loft was safe for protection of invitees who entered the nightclub. Here, however, the man's injury was not caused by an unsafe condition in the nightclub loft. Rather, the man was injured when the investor tossed the man out of the club and onto the street. As such, this answer is incorrect.

If you know someone, then you are privileged to deliver more force or threat than usual to protect them from assailant.

The correct answer is:Yes, because the law student was privileged to threaten use of deadly force if reasonably necessary to save the classmate's life. Discussion of correct answer:The law student is privileged to threaten more force than she is privileged to deliver. The defense of privilege may be used to shield a defendant from liability if some important personal or public interest will be protected by the defendant's otherwise prohibited conduct and this interest justifies the harm caused or threatened. In this case, the law student is privileged to threaten deadly force, because she is attempting to save the classmate's life. The interest of saving a human life justifies the law student's threat. Incorrect. No, unless there was a statute specifically granting such a privilege. The privilege exists at common law and need not be embodied in a statute to be available to the law student. Thus, this answer is incorrect.

it is the friend's intentional action in borrowing the car without permission that dispossessed the motivational speaker of her car and caused actual damages that gives rise to the cause of action trespass to chattel and actual damages.

The correct answer is:Yes, because the motivational speaker suffered actual damages from the intentional dispossession. Discussion of correct answer:A trespass to a chattel may be committed by intentionally dispossessing another of the chattel, or by using or intermeddling with a chattel in the possession of another. The plaintiff must prove actual damages as an element of the trespass to chattels cause of action. Here, the friend borrowed the motivational speaker's car, an intentional act that dispossessed the motivational speaker of her car when she needed it. Given that the motivational speaker lost a business client and his large account as a result of the friend's dispossession of the vehicle, it appears that the motivational speaker will be able to prove actual damages. Thus, the elements of the tort are present, and the motivational speaker is likely to prevail in her claim. Incorrect. Yes, because the friend should have anticipated that the motivational speaker might return home early. It may have been smarter for the friend to anticipate that the motivational speaker might return home early and need the car, but the lack of this anticipation is not what gives the motivational speaker grounds for a trespass to chattels action. Instead, it is the friend's intentional action in borrowing the car without permission that dispossessed the motivational speaker of her car and caused actual damages that gives rise to the cause of action. Therefore, this is not the best answer choice.

Even if your property does not intentionally enter onto the land, leaving it there is trespass.

The correct answer is:Yes, because the neighbor failed to remove the kite. Discussion of correct answer:One is subject to liability to another for trespass to land, even without harm to another's legally protected interest, if he: (1) intentionally enters land in the possession of the other or causes a thing or a third person to do so; (2) remains on the land; or (3) fails to remove from the land a thing that he is under a duty to remove. Here, given that the wind caused the kite to fly onto the estate, the neighbor did not intentionally cause the kite to enter the estate. However, since the neighbor owned the kite, he had a duty to remove it from the property upon the farmer's request. Because he failed to remove from the land a thing that he was under a duty to remove, he will be liable for trespass to land. Discussion of incorrect answers: Incorrect. Yes, because the neighbor's kite is a tangible physical object that entered the estate. Historically, trespass required the entry of some tangible physical object onto the property of another. Modern courts are extending trespass liability to intrusions caused by microscopic particles and invisible gases, if harm is caused. Therefore, while it is true that the farmer will most likely prevail in a trespass action against the neighbor, it is not merely because the kite is a tangible object. Given that the wind caused the kite to fly onto the estate, the neighbor did not intentionally cause the kite to enter the estate. As such, the neighbor was not liable for trespass merely because his kite entered property in the farmer's possession. This answer neglects to note that it is the neighbor's subsequent failure to remove the kite on the farmer's request that makes the neighbor liable for trespass.

A Battery is an intentional act that causes a harmful or offensive contact. This can include coat tearing.

The correct answer is:Yes, because the pickpocket tore the man's coat pocket while stealing his wallet. Discussion of correct answer:A battery is an intentional act that causes a harmful or offensive contact with the plaintiff or with something closely connected thereto. It need not be direct contact with the plaintiff's body -- offensively touching anything close to or connected to the plaintiff's body is enough. Here, the pickpocket tore the man's pocket, a touching that a reasonable person would consider offensive and that was close to and connected to the man. Therefore, although the pickpocket did not harm the man's body, the pickpocket is liable for battery. Incorrect. No, because the pickpocket did not physically harm the man. Physical injury is not an element of battery. A person commits battery if he acts intending to cause a harmful or offensive contact or an imminent apprehension of such a contact, and an offensive contact directly or indirectly results. It need not be direct contact with the plaintiff's body -- offensively touching anything close to or connected to the plaintiff's body is enough. Here, the pickpocket tore the man's pocket, a touching that a reasonable person would consider offensive and that was of something close to and connected to the man. The fact that the pickpocket did not physically injure the man will not relieve him of liability for the tort of battery.

If a defendant has reason to know someone might be trapped by a locked door, even if there are alternative exits, then its a sign of false imprisonment.

The correct answer is:Yes, because the principal knew that there would likely be parents in the building. Discussion of correct answer:If the defendant knows with substantial certainty that his actions will cause confinement of a person, the requisite intent is achieved. The defendant need not know the identity of the particular plaintiff. In this case, parents were observing classes almost every day, so the principal should have known that parents were likely to have been there. Incorrect. No, because only one door was locked and there were several other exits. An action will not lie for false imprisonment where the defendant merely impedes the plaintiff's progress in one particular direction, or confines him within an area from which there is an obvious, reasonable exit. In this case, the plaintiff's confinement is total; although other exits exist, she was not aware of them. No reasonable exit was apparent. Also, the facts imply that the other exits would not actually allow the mother to leave the school grounds, because it is stated that they lead to a fenced-in schoolyard. Thus, this answer is incorrect.

Making a statement of fact not knowing all relevant facts is a misrepresentation (act with scienter)

The correct answer is:Yes, because the professor was unaware whether the graduate had been convicted of a crime when she answered the employment director's inquiry. Discussion of correct answer:There are five elements required to establish a claim for misrepresentation: (1) there must be a material misrepresentation; (2) the defendant must act with scienter; (3) the defendant must intend to induce reliance; (4) the plaintiff must justifiably rely on the misrepresentation; and (5) the plaintiff must suffer pecuniary damages. The key to this question is understanding the element of scienter. This element is satisfied in two situations: (A) when the defendant lies; or (B) when the defendant makes a statement of fact not knowing whether it is true or false. Here, the professor stated that the graduate had never been convicted of a theft crime even though she was aware that she did not know the answer to this question. Therefore, the professor acted with scienter when she stated that the graduate had never been convicted of a theft crime. Choice (C) is not correct because, while much of the professor's statement consists of opinion that would not be actionable, the professor stated a fact when she asserted that the graduate had never been convicted of a crime. Choice (D) is incorrect because the requirement is that the defendant act with scienter, and intentionally lying is just one way of satisfying this element. Incorrect. No, because the professor did not intentionally lie to the employment director.

Defamation requires showing a disregard for the truth (not negligence).

The correct answer is:Yes, because the reporter believed the story to be true. Discussion of correct answer:Defamation of a public official requires the plaintiff prove that the defendant acted with actual malice: intentional knowledge of falsity or a reckless disregard of the truth. Negligence--which is what is present in this case--is insufficient as a First Amendment matter to impose liability [New York Times v. Sullivan, 376 U.S. 254 (1964)]. Incorrect. No, because the reporter was negligent. Negligence in the defamation of a public official is actually exculpatory rather than a basis for liability.

Conversion is more substantial than trespass to chattel. If you take something away for a long period of time, it is converted.

The correct answer is:Yes, because the student converted the moped. Discussion of correct answer:Conversion requires an intentional exercise of control and dominion over another person's property that results in the destruction of the property or a substantial interference. A plaintiff can elect to recover damages or the return of the property. The usual measure is the fair market value of the property. The requirement of dominion and control is satisfied because the student intentionally took the bike for two months. During that time the bike was used and damaged. The facts support the conclusion that there was a substantial interference with the classmate's right to immediate possession of the moped. Incorrect. Yes, because the student has committed a trespass to chattel. This answer is incorrect because trespass to chattel is more appropriate when there is a less substantial interference or a dispossession of much shorter duration. Because there was a deprivation for two months and a substantial interference, the appropriate cause of action would be conversion rather than trespass to chattel.

If a model did not give her consent to use her likeness, then it could not be used. Reasonable justification by unknowing user doesn't matter. The tort of appropriation of identity or likeness applies when a defendant uses another's name or likeness for commercial advantage without authorization. Mistake no defense.

The correct answer is:Yes, because the supermodel did not give permission to use her likeness. Discussion of correct answer:One who uses without authorization the name or likeness of another for commercial advantage (to promote a product or service) is subject to liability for the invasion of privacy tort of appropriation of identity or likeness. There is no intent element to this tort, so it is irrelevant that the pharmacist believed that the supermodel's photograph in the display was authorized. Here, all of the elements of appropriation of identity are present, so the supermodel will prevail in her claim. Incorrect. Yes, because the pharmacist was not reasonably justified in believing that his use of the supermodel's photograph was authorized. The tort of appropriation of identity or likeness applies when a defendant uses another's name or likeness for commercial advantage without authorization. This answer implies that if the pharmacist's reliance on the saleswoman's representations that the supermodel had given permission was reasonable, he would not be liable. In fact, however, mistake as to whether consent was given, even if this mistake is reasonable, is not a valid defense to the tort.

Offensive contact can be transferred for purposes of a battery.

The correct answer is:Yes, because the surgeon's intent to make offensive contact with the scrub nurse was transferred to the actor. Discussion of correct answer:The tort of battery occurs when a defendant intentionally causes harmful or offensive contact with the plaintiff, or something closely connected to the plaintiff. That intent can be transferred where the defendant intended harmful or offensive contact against one person but the resulting harm is caused to the plaintiff. Here, the surgeon intentionally slapped the hand of the nurse, resulting in harmful contact with the actor. The surgeon's intent to make offensive contact with the scrub nurse is transferred to the actor, and the surgeon's action constitutes battery not only on the nurse but also on the actor. Discussion of incorrect answers: Incorrect. Yes, because the actor suffered an injury, albeit a temporary injury, when the surgeon slapped the scalpel out of the nurse's hand. This answer choice is incorrect, because it gives the wrong reason why the actor has a claim in battery against the surgeon. Physical injury is neither necessary nor sufficient to prevail in a battery claim. Battery (in tort) occurs when a defendant intentionally causes harmful or offensive contact with the plaintiff, or something closely connected to the plaintiff. Intent may be transferred in cases where the defendant intends the harmful or offensive contact against one person but the resulting harm is caused to the plaintiff. Here, the surgeon intentionally slapped the hand of the nurse, causing harmful contact with the actor. The surgeon's intent to make offensive contact with the scrub nurse is transferred to the actor, and the surgeon's act constitutes battery not only on the nurse but also on the actor.

Get on the sidelines where players tackle fans and get attention and you basically consented. Doesn't help if you shouted out "come get it" because the other person is not obliged to read your mind.

The correct answer is:Yes, because the woman consented to the contact. Discussion of correct answer:A defendant will not be liable in a battery action if the plaintiff consented to the contact in question. Consent is defined as a plaintiff's willingness in fact that the tortious contact take place. Consent may be granted expressly, in words, or it may be implied from the plaintiff's conduct. Here, as a regular watcher of neighborhood football games, the woman was clearly aware of the neighborhood tradition whereby players, after making a touchdown, would tackle fans on the sidelines. As such, it is arguable that merely by attending the game, the woman impliedly consented to such contact. However, even if the woman is not found to have impliedly consented to being tackled, the facts indicate that she overtly called out to the player, "Come on, big boy!" By her words, the woman expressly consented to being tackled. The woman's battery claim will fail. Incorrect. No, because the woman did not expressly consent to the player's contact. A defendant will not be liable in a battery action if the plaintiff consented to the contact in question. The woman both consented impliedly and expressly. She was aware of the local tradition of players tackling fans after a touchdown, so by attending the game she arguably gave implied consent to the contact. She also gave express consent when she called out to the player after the touchdown. Thus, this answer is incorrect.

Defamation vs Invasion of Privacy: defamation is insulting, never a "compliment/lie" - otherwise it is just a "false light" issue and therefore invasion of privacy.

The correct answer is:Yes, for invasion of privacy only, because the book falsely portrayed Marvel's baseball career. Discussion of correct answer:Here's an extremely difficult Multistate example that requires students to differentiate between defamation and invasion of privacy. Although Kosale falsely depicted Marvel's accomplishments, he did not really defame him. Defamatory material is that which tends to lower the reputation of a person by impugning his honesty, integrity, or sanity, or by diminishing the esteem, respect, or confidence in which the person is held. By enhancing Marvel's exploits, Kosale did not lower or diminish his reputation. On the other hand, Kosale would be liable for invasion of right to privacy by casting Marvel in a "false light." In order to establish a prima facie case for "false light," the following elements must be proven: (1) publication of facts that place the plaintiff in a false light; (2) the false light must be objectionable to a reasonable person; and (3) malice on the part of defendant where the published material is in the public interest. Incorrect. Yes, for defamation only, because some of the representations of Marvel's career were known by Kosale to be fictional and were likely to be offensive to Marvel.

If they did not understand what you said to the other guy, it never happened for the sake of Defamation.

The correct answer is:Yes, if one or more of the other members of the dance company understood English. Discussion of correct answer:For spoken words to be defamatory and constitute slander, there must be publication. Publication of slander occurs only when a third party hears and understands the words (so that the hearer may have a lower opinion of the plaintiff). If they were spoken in a language none of the listeners understood, they were not "published." Thus, this statement is actionable only if one or more of the persons who heard it understands English. Discussion of incorrect answers: Incorrect. Yes, because the female skater's statement constituted slander per se. It is true that these words impute a loathsome disease to the plaintiff, which makes the statement slander per se. However, this answer is incorrect, because the words are only actionable if they were "published." Because it is unclear whether anyone understood the statement, publication is the key issue.

Official rescuers can be held accountable for negligent actions (such as properly fastened straps),

The correct answer is:Yes, if the harness latch was not properly fastened to the stretcher. Discussion of correct answer:Under the facts presented, the hiker's cause of action against the rescuer and the mountain rescue team would be in negligence. (Inasmuch as neither the rescuer nor the mountain rescue team were in the business of selling harnesses, the hiker would not have a strict product liability claim against them.) Because the rescuer worked for the mountain rescue team, the hiker would be able to recover against them if either the rescuer or the mountain rescue team were negligent. The rescuer and the mountain rescue team had a duty to act towards the hiker according to the appropriate standard of care in order to protect her from unreasonable risk of injury. Because the rescuer and the mountain rescue team were professional "rescuers," they would, at a minimum, be held to a reasonable person standard and, at a maximum, be required to exercise the knowledge and skill of a member of their profession in the same or a comparable community. If the harness latch was not properly fastened to the stretcher, they would have failed to meet the appropriate standard of care and would incur liability to the hiker for negligence. Thus, this is the best answer. Incorrect. No, if the hiker has a sustainable claim against the harness manufacturer. The hiker would not be prevented from recovering from the rescuer and the mountain rescue team merely because she is able to recover in a strict liability suit from the harness manufacturer. Thus, this answer is incorrect.


Related study sets

Chapter 8, 9, 17, 18 Study Guide

View Set

Chapter 8 Articulations Mastering

View Set

Psychosocial Nursing: Exam 2 Practice Questions

View Set

Ch.10 Assessing for Violence PrepU

View Set

ACCT 2101 (Chenier) Chapter 1 Connect HW

View Set